AT 3312 pp exam 1 material, AT 3312 exam2 corrections, clin 2 final, clin 2 pract, txst AT 3312 exam 2, last set, AT 3312 Final

Pataasin ang iyong marka sa homework at exams ngayon gamit ang Quizwiz!

external coxa sultans "snapping hip syndrome"

either the posterior border of the ITB or the anterior border of gluteus Maximus snapping over the greater trochanter of the femur, common among repetitive athletes, more common of the two types

The ligamentum flavum has a higher concentration of ___________________ than any other ligament in the human body.

elastin

atypical SCFE

most likely to develop in older children and adolescents, from age 9 until the end on growth, much more common in boys, chances are greater if patient is overweight or obese, slipping at the femoral neck growth plate may be sudden or gradual

the middle part of the achilles tendon has ______ than either the proximal or distal park, predisposing the waist of the tendon to __

much lower vascular density, degeneration and spontaneous rupture

factors involving scheuermanns disease

multiple vertebrae are affected, most commonly occurs between T10 and L2

dynamic stabilizers of the knee

muscles

Somatic (body) pain originates from

muscles, soft tissues and skin

According to the Loudon chapter in the Reiman (2016) text, the loading response subphase of the stance phase of gait is marked by which one of the following visible events?

foot flat

what the clinician sees during initial contact

foot makes initial contact with ground, in 85% if the population, the lateral heel makes this initial contact

extrinsic risk factors for tibial stress fracture

footwear, training methods and surfaces

most gait analysis today is performed with ______ to measure ground reaction forces, _________ to measure muscle activity and _________ to measure movements (Magee)

force platforms, electromyography (EMG), speed motion analysis systems

talar avascular necrosis (AVN) results from trauma to the

fragile blood supply to the talus

Which one of the following surgical techniques is most effective for repair of large (~8 cm squared) hyaline articular cartilage lesions on the medial or lateral femoral condyle?

fresh osteochondral allograft

a wedge shaped talus makes _________________ at the talocrual joint

full dorsiflexion close packed position

treatment for scheuermanns disease

full time use of a brace (20 hrs/day) is usually required initially until maximum correction has been achieved

5th cardinal sign of inflammation

function lasea (loss of function)

leg length difference that is attributed to something other than the length of the tibia or femur, possible causes: tightness of muscle or joint structures or muscular weakness in the lower extremity or spine; examples include knee hyperextension, scoliosis or pelvic muscle imbalances

functional or apparent leg length

the motion segment is the ____ composed to two adjacent vertebra, the intervertebral disc, connecting ligament, and four facet joints and capsules

functional unit of the spine

superficial posterior tibial compartment muscles

gastrocnemius, soleus, and plantris

all plantar flexors

gastrocnemius, soleus, fibularis longus, fibularis brevis, tibialis posterior, flexor digitorum longus, flexor hallucis longus and plantaris

talocrural joint movements in the sagittal plane = plantar flexion

gastrocnemius, soleus, tibialis posterior, fibularis longus, fibularis brevis, FDL, FHL and plantaris

Which of the following muscles, when acting concentrically, produce external rotation at the hip? (Please read this question carefully and identify all the correct answers).

gemellus superior quadratus femoris piriformis obturator internus gemellus inferior obturator externus

One of the most controversial dysfunctions in orthopedic medicine....

generally accepted that approximately 13% (the 95% confidence interval range is from 9-26%) of patients with persistence low back pain have the SIJ as the confirmed orgin of that pain

low back pain etiology

genetics, mechanical loading poor flexibility, poor core muscular endurance, muscle asymmetries, improper kinematics

motor= cremaster and sensory= scrotum in males and mons pubis in females

genitogemoral nerve L1, L2

at initial contact, the thorax is thrust posteriorly to maintain hip extension during the stance phase, often causing a lurching of the trunk

gluteus maximus gait, caused by weakness or paralysis of the gluteus Maximus

A patient who demonstrates a Trendelenburg gait indicates that they have a weakness or paralysis of which one of the following muscles?

gluteus medius

A positive Trendelenburg test indicates a significant weakness or paralysis of which of the following muscles? (Please read this question carefully and identify all the correct answers).

gluteus medius

Apophysis avulsion fracture of greater trochanter muscles involved

gluteus medius and minimus

Primary hip abductor muscles

gluteus medius, gluteus minimus, gluteus maximus, tensor fasciae latae, sartorius, piriformis, rectus femoris (secondary)

validity uses the concept of

gold standard

orthopedic special tests that have very high specificity rates:

good for ruling in, a positive test will rule in because there are very few false positives

orthopedic special tests that have very high sensitivity rates:

good for ruling out, a negative results effectively rules out because there are very few false negatives with special tests with very high sensitive

the meyerding scale is based on quartiles (25%) of anterior displacement of the superior vertebra on the inferior vertebra

grade 1=0-25% anterior displacement or slippage up to grade 4 76-99% slippage

foot types

greek=22% egyptian=69% and square=9%

3 possible outcomes of triage

green flag; I will treat yellow flag; I will treat then refer red flag; I will immediately refer

physics vs epiphysis vs apophysis

growth plate vs rounded end vs secondary growth center

activation of the ________ assists (agonist) the ACL's efforts to limit _________ by creating knee flexion torque, pulling the tibia posteriorly

hamstring muscles, anterior tibial translation

Which of the following are dynamic stabilizers that act(s) as an agonist(s) to the anterior cruciate ligament (ACL) and help(s) resist anterior tibial translation? (Please read this question carefully and identify all the correct answers).

hamstrings

A postive J sign

having patient extend knee form 90 degrees flexion to full extension, positive indicates a laterally tracking patella

Which one of the following is NOT a physical sign of injury?

headache

Hip osteoarthritis clinical presentation

- hip pain, hip internal rotation ROM of >_ 15 who also experience pain in internal rotation -Morning stiffness for _< 60 min - Age 50 or older Sensitivity (true positive rate) of the 3 criteria .86

overuse injuries in runners, impact peak can range from

1.5 BW to 5 BW

If the medical diagnosis is delayed, or if the treating clinician decides to treat and refer, then the CORRECT DIAGNOSIS IS EVENTUALLY MADE when 1 of 3 things occurs:

1.The patient does not get better with physical medicine intervention... 2.The patient gets better, then gets worse... 3.Other associated signs and symptoms develop...

medical evidence hierarchy

meta-analyses/ systematic reviews, randomized clinical trials, clinical trials, cohort studies, case control studies, case series (several), case report (single patient) and expert opinion

The perimeniscal capillary plexus creates the "red-red zone" by supplying blood to the outer ________________ percent of the medial and the lateral menisci.

25 to 30

ankle sprain are one of the most common musculoskeletal and sports related injuries, constituting nearly _______ of all musculoskeletal trauma cases and almost ___ of all sports related trauma cases

25%, 40%

how many bones are in the foot

26

most common fracture with greek (Mortons) foot type

2nd metatarsal stress fracture

How many of the cardinal planes of motion (or cardinal axes of motion) are possible at every lumbar zygoapophyseal ("facet") joint?

3

what planes does the hip move in

3 cardinal planes: sagital, frontal and transverse

lisfranc joint

3 cuneiforms, cuboid and 5 metatarsals

When reading published research in physical medicine, which one of the following types of studies is considered to yield/produce the "best" (highest quality) evidence?

meta-analysis

MRI cons

metallic fragments in the eyes of patients, patients with pacemakers, and aneurysm clips in the brain

Involved one or more of the metatarsal heads, common among ballet dancers, runners and jumpers. Typically treated conservatively and resolves without surgery

metatarsalgia (pain in the metatarsals)

The component of the stance phase of gait that occurs then the body's center of gravity is directly over the foot and the whole foot is in contact with the ground is known as __________________________.

midstance

an ankle sydesmosis injury is a _________ than a lateral ankle sprain

more serious injury

footwear

most important characteristic is shock absorption. shoes lose ability to absorb shock after 300-500 miles

3 fascial (muscle) compartments of the thight

Anterior: Hip flexors and knee extensors ( FEMORAL N) Medial: Hip adductors and knee internal rotators (OBTURATOR N) Posterior: Hip extensors and knee flexors (SCIATIC N)

the symptomatic narrowing of the spinal canal or intervertebral foramen, will produce either unilateral (spinal nerve root) or bilateral (spinal cord) leg pain

spinal stenosis

The sacroiliac joint (SIJ) serves as the supporting base of the _____ and as a

spine, point of intersection between the spinal and lower extremity joints

examples of mechanical low back pain with associated neurogenic leg pain. (7-10% of all LBP and typically caused by radiculopathy)

spinal stenosis, herniated nucleus pulposus, spondyloisthesis

plates of cartilage that cover almost all the entire superior and inferior aspects of the disc and bind it to their respective vertebral bodies

superior and inferior vertebral endplates

motor= gluteus maximus, gluteus medius and tensor fascia lata and sensory= non

superior gluteal nerve L4, L5, S1

lauge-hansen classification system. 4 ankle fracture types that are based upon the position of the foot and deforming force at the time of injury

supination-adduction, supination-external rotation, pronation-abduction and pronation-external rotation

hypothesis: relative effort during walking would be hight among older women compared with younger women

supported, alderwomen exerted higher relative effort in hip flexion and knee extension than younger women

hypothesis 1: relative effort during walking would be higher among women who are obese compared to healthy weight women

supported, relative effort during walking was 20-40% higher among obese women

iliotibial band servers as tendon of insertion of

tensor fasciae latae (anteriorly) and the gluteus Maximus (posteriorly) in a level distal to the great trochanter. Lateral stabilizer of the knee

mechanics

the branch of science that deals with the effects of forces and energy on bodies

interrater (interexaminer) reliability

the extent to which different examiners obtain the same results on the same patients

Intrarater (intraexaminer) reliability

the extent to which the same examiner obtains the same results on the same patient

supination-external rotation

the first structure to rupture is the anterior tibiofibular ligament followed by a spiral fibula fracture, posterior malleoli fracture than medial malleolus fracture/ or deltoid ligament rupture (most common)

case study -- the knee is flexed to 90 deg, and the hip is hyperextended

the gluteals

what the clinician sees during terminal stance

the heel comes off the ground

reliability

the level of agreement between test and retest

Meniscal injury MOI - exam 2

the medial meniscus has attachments to the deep medial collateral ligament and is exposed to injury w the application of valgus stress

disc height is preserved with age but is dependent on

the nucleus pulposus

Stable slipper capital femoral epiphysis

the patient can walk or bear weight on the affected hip, either with or without crutches. most cases are stable

Unstable Slipped Capital Femoral Epiphysis

the patients cannot walk or bear weight even with crutches. requires urgent treatment, complications are much more common

interpretations of positive and negative likelihood ratios are good when

the positive liklihood ratio is over 10 and the negative likelihood ratio is less than .1

clinical significance of patella Alta

the presence of patella Alta (high patella) in the group of athletes with chronic tendinopathy of the knee extensor mechanism was significantly higher than in the control group

Triage

the process of determining the priority of treatment based on the severity of the injured persons condition

kapp coefficient

used if the test result is a positive or negative or yes/no. Kappa values range from 1.0 to 0.0

intraclass correlation coefficient (ICC)

used if the test result is an interval or continuous scale value with a hand held dynamometer, ICC values range from 1.0 to 0.0

positive and negatives to radiography

uses ionizing radiation, widely available, relatively low cost, not much use beyond identifying fractures

most common MOI of medial collateral ligament

valgus producing force with foot planter=d, severe hyperextension of the knee

Patient-reported outcome (PRO) measures, e.g., Roland-Morris Disability Questionnaire, as discussed in the Reiman (2016) text, are considered ____________________ if they measure what they are intended to measure.

valid

most common MOI of lateral collateral ligament

varus producing force with foot planted and severe hyperextension of the knee

The _____________________________________ is the most appropriate test to specifically evaluate the status of the fibular collateral ligament of the knee.

varus stress test at 30 degrees of knee flexion

the lateral collateral ligament is the primary restrain to _______ loads; ___ at 5 degrees of flexion and __ at 25 degrees of flexion

varus, 55%, 69%

red on red zone

vascularized outer 1/3 of the meniscus, best healing potential

According to Reiman (2016), which of the following are characteristics demonstrated by "good diagnosticians" during the history-taking interview with patients, and are typically observable within the first 3 minutes of the encounter? (Read carefully and be certain to indicate all correct answers).

verifying information obtained from the patient thoroughness of inquiry regarding the chief complaint establishing a rapport with patient

the absence of vertical or horizontal trabeculae with osteoporosis diminishes a

vertebra's capacity to resist compressive loads

the line of gravity is always

vertical

repairable meniscal tears

vertical longitudinal and radial(possibly)

pathomechanics of osteitis pubis

overload or training errors. Great deal of running, changing direction, weight bearing hard surfaces, uneven ground, beginning after long lay off period, increasing intensity or duration too quickly

tarsometatarsal glides test

pos -- laxity imp -- internal foot ligament sprain or joint adhesion

midtarsal joint glides test

pos -- laxity and/or pain imp -- lisfranc sprain

Dorsiflexion-Eversion Test

pos -- neurological reproduction of symptoms imp -- tarsal tunnel syndrome

Tinel's Test (ankle)

pos -- numbness/tingling/burning imp -- nerve pathology

squeeze test

pos -- pain imp -- gross or stress fracture of the fibula, syndesmosis sprain

McConnell Test

pos -- pain and alleviation of pain imp -- patellar chondromalacia

Anterior Drawer Test (ankle)

pos -- pain and ant. translation of ankle imp -- grade 1-3 ankle sprain

Cotton Test (Lateral Talar Glide)

pos -- pain and increased lateral translation of the talus imp -- syndesmosis sprain

ege's test

pos -- pain and/or apprehension imp -- meniscal tear

Mulder Sign

pos -- pain and/or tenderness imp -- intermetatarsal neuroma

Apley Compression/Distraction Test

pos -- pain during compression, that is reduced during distraction imp -- meniscal tear for BOTH

external rotation stress test (kleiger)

pos -- pain in medial joint or anterolateral ankle imp -- medial pain is damage to deltoid ligament, AL pain is syndesmosis pathology

noble compression test

pos -- pain on IT band or snapping imp -- IT band friction syndrome

crossover test ALRI

pos -- pain or instability imp -- Instability of the lateral capsular restraints

crossover test amri

pos -- pain or instability imp -- Instability of the medial capsular restraints

Clarke's Sign (Patellar Grind Test)

pos -- pain, secondary signs are grinding/popping imp -- OCD or chondral lesions under the patella

Homan's sign

pos -- pain/tenderness imp -- DVT

McMurray Test

pos -- popping/clicking/locking of knee imp -- meniscus tear or meniscal damage

godfreys test

pos -- post displacement of tib tuberosity imp -- pcl sprain

nonsurgical treatment for medial ankle sprains

protect the injured plane with a brace and promote movement in the unaffected plane of motion

nonsurgical treatment of lateral (inversion) ankle sprain

protect the injured plane with an ankle brace and promote safe movement in the unaffected plane of motion

design features of the lumbar spine

provides axial rigidity to the abdominal portion of the trunk, separates the thorax from the pelvis, enables movement between the thorax and pelvis, provides sites of origin for abdominal muscles and insertion for the latissimus doors

limits hip abduction and hyperextension, just medial to the iliofemoral ligament

pubofemoral ligament

motor= urethral and anal spinchter muscles and sensory= pensis, clit, and perineum

pudendal nerve S2, S3, S4

key vital signs

pulse rate, respiration rate, blood pressure, and temperature

a 4 sided muscle in the lumbar region

quadrates lumborum

______ activation acts against (antagonist) the ACL and pulls the tibia forward, loading the ACL

quadriceps

disease of spinal nerve root, possible weakness or numbness

radiculopathy

With regard to the evidence-based practice of physical medicine, which one of the following experimental designs is considered to be the "gold standard" (best) in medical research?

randomized controlled trial

primary ACL non contact injury mechanisms

rapid deceleration with a change of direction, off balance or awkward landing from a jump with the knee in valgus with little knee flexion, ACL injury typically occurs during one legged landings

chronic (exertion) tibial compartment syndrome is

rarely a medical emergency and is caused by repetitive stress during exercise

calcaneus angled away from midline (foot in everted or pronated)

rearfoot valgus

calcaneus angles toward the midline (foot is inverted of supinated)

rearfoot varus

Which of the following are included among the "red flag findings" identified by Reiman (2016) that warrant immediate referral of the patient to physician? (Read carefully and be certain to indicate all correct answers).

rebound tenderness fatigue or malaise increased urination or thirst night sweats shortness of breath

Which one of the following muscles is typically inhibited in patients who present in clinic with Janda's "lower crossed syndrome"?

rectus abdominis

which of the following tendons are pulled away from bony origins - exam 2

rectus femoris with an apophyseal avulsion of the AIIS

individuals with chronic ankle instability often report

recurrent sprains and giving way sensations at the ankle joint

us army basic combat training phases

red (1-3weeks) white (4-5 weeks) blue (6-9)

triage screen and flag color

red=refer yellow=adverse prognostic indicators blue=perceptions about the relationship between work and health black= professional culture, health care policy, the insurance environment that the clinician must operate in

patellofemoral pain syndrome treatment

reduce inflammation, NSAIDs, improve extensor mechanism muscular strength and balance

etiology of low back pain

regional mechanical LBP=90% mechanical LBP with neurogenic leg pain=7-10% non-mechanical spine disorders=<1%

no neurological symptoms accompany

regional mechanical low back pain

knee muscles are activated via the feedback system to

regulate the dynamic stability of the knee and possibly prevent ligament injury

at night, when the spine is _______, the discs elevated swelling pressure _______ from surrounding tissues

relatively unloaded, draws in water

relaxin and pregnancy

relaxin decreases stiffness in cartilage, joints and tendons and creates laxity in connective tissue, as pregnancy progresses the amount of relaxin increases to allow growth of the baby and create more laxity within the pelvis to facilitate an easier delivery

The human ACL has ________ receptors and female collegiate athletes with ___________ concentrations above 6 have been shown to have more than _____ times increased risk for ACL tears

relaxin, serum relaxin, 4

abnormal growths of new tissue called neoplasms and relatively rare. Primary tumor originates within the bony structures of the spine and secondary tumor originates in a part of the body other than the spine but hone spreads to the spine

spinal tumors

Kyphosis

An excessive posterior curvature of the thoracic spine (Hunchback)

Milgram Test

- Pos - pain or can't hold legs up for the time - Ind - herniated disk, nerve root impingement

Slump test (with variations

- Pos - sciatic pain or neurological symptoms - Ind - sciatica or nerve root impingement

Valsalva Test

- Pos -- pain/ neurological symptoms they have (ex- numbness/tingling/burning) Ind-- herniated disk, or lumbar nerve root impingement

what muscles allow for hip abduction in MMT

- glute med - glute min - glut max - tensor fascia latae - sartorius - piriformis

non operative treatment of achilles tendon ruptures

-Most recommend fiberglass casting with the foot in equinus (plantar flexed position) for a period of 8 weeks. -Once the cast is removed, active and passive motion exercises are begun. -Some orthopedic surgeons will continue to protect the tendon with a brace (walking boot) for 4 more weeks...

clinical pearls for Ottawa rules

-Palpate the entire distal 6 cm of the fibula and tibia -Do not neglect the importance of medial malleolar tenderness -"Bearing weight" counts even if the patient limps -Be cautious in patients under age 18

Osgood-Schlatter disease

-a form of osteochondritis that is caused by repetitive traction on the tibial tuberosity -most common in physically active boys (12-15) and girls (11-13) -prominent tibial tuberosity activity restrictions, and pain on the kneeling are the classic signs and symptoms

knee osteoarthitis in America

-affects most adults over 65 -has higher prevalence than all other types of OA -women (42%) are more affected than men (31%) -obesity increases risk

Apophyseal avulsion fracture

-almost exclusive to the athletic adolescent population within 11-17 yr old -occur most often from large tensile forces created during running or sprinting

singing Larsen Johansson syndrome

-an irritation of the immature inferior pole of the patella -affects a slightly younger population than Osgoods (10-12) -high correlation between the syndrome and running/ jumping activities -also know as self limiting condition

talocrural joint movements

-average AROM in dorsiflexion= 20 -10 of dorsiflexion required for efficient gait and 20 required for descending stairs -average AROM for plantar flexion is 45-50 -20-25 of plantar flexion necessary for efficient gait

ottawa ankle and foot rules .A foot x-ray series is required if there is any pain in the midfoot zone and any one of these findings

-bone tenderness at the base of 5th metatarsal -bone tenderness of the navicular -inability to bear weight both immediately and in the emergency department (4 steps!)

Ottawa ankle and foot rules. An ankle x-ray series is required if there is any pain in the malleolar zone and any one of these findings

-bone tenderness on the posterior edge or tip of the lateral malleolus -bone tenderness on the posterior edge or tip of the medial malleolus -inability to bear weight both immediately and in emergency department (4 steps !)

myositis ossificans traumatic of the femur

-calcifications typically occur within 2 to 6 weeks after onset of symptoms -absence of calcification at this time makes the diagnosis of myositis ossificans less likely

Osteomyeltis of the hip (an infection of the bone)

-caused by bacteria called staphylococcus aureus -affects about 2/10,000 rare but serious -in adults, often affects the vertebrae and the pelvis - in children, usually involves the femur and tibia

athletic pubalgia "sports hernia"

-chronic groin and lower abdominal pain caused by an incompetent abdominal wall -often occur where the abdominals and adductors attach at the pubic bone -traditional hernias occur from a defect in the inguinal ring in the inguinal canal

Clinical examination of posture using high tech tools

-cobb angles from radiographs to quantify the severity of kyphosis and scoliosis -digitized still photography with Biotonix system(4 images) -3D kinematic analysis using computer software

genetic inheritance accounts for

-degenerative disc diseased 50-75% -osteoarthritis 60% -low back pain60%inwomenand30-46%inmen

menisci functions

-distribute stresses across the knee during weight bearing activites -absorb shock -provide secondary joint stabilization -facilitate hyaline articular cartilage nutrition and lubrication -facilitate joint gliding -prevent knee hyperextension -protect the joint margins

MOI achilles tendon rupture

-eccentric loading of a dorsiflexed ankle with the knee extended, putting soleus and gastrocnemius on maximal stretch -overuse or overtraining -poor or nonexistent stretching regimens -direct trauma -tight calf muscles -structural deficiencies

vascular screening

-evaluate the major pulses in the region of the affected body part -check capillary refill -observe skin color -note the temperature of the skin

Counternutation is sacral extension and involves:

-extension of the sacrum relative to the ilia -superior aspect of the sacrum moving posteriorly -stresses the long dorsal ligaments

retinacula at the ankle

-extensor renticula (superior and inferior) -fibular renticula (superior and inferior)

Nutation is sacral flexion and involves:

-flexion of the sacrum relative to the ilia -superior aspect of sacrum moving anteriorly -stresses the sacrotuberous ligament

Femoroacetabular Impingement (FAI)

-found in young and middle aged adults, typically men -insidious onset groin pain may be preceded by minor trauma -in early stages the pain is intermittent and may be exacerbated bu physical activities and exercise -often misdiagnosed was hip or groin pain -prolonged sitting may be painful

tibial compartment syndrome signs and symptoms

-general, non-specific leg pain -pain in region corresponding to a specific tibial compartment -paresthesis over the affected compartment, radiating into the foot -aching or cramp-like pain

kellgren and Lawrence classification system of knee osteoarthritis severity

-grade 0= no radiographic evidence -grade 1= no joint space narrowing but possible osteophytic lipping -grade 2= definite JSN and osteophytes -grade 3= moderate and multiple osteophytes, definite JSN and sclerosis with possible bony deformity -grade 4= most severe, large osteophytes, marked JSN, severe sclerosis and definite bony deformity

conditions that warrant terminations during ligament testing, vascular test and neurological test

-gross joint instability, grade 3 -diminished or absent pulse -neurological dysfunction, sensory dysfunction, absent or dimisnished reflexes, motor dysfunction

most common MOI of posterior coruscate ligament

-hyperFLEXION of the knee -dashboard injuries -severe hyperextension -large varus or valgus producing forces with foot planted -any of the above combine with large torsional force at the knee

3 components of the swing phase

-initial (early swing occurs immediately after toe off and involves acceleration of the limb forward) -midswing (middle portion is called forward swing/ swing through; the toes clear the ground) -terminal (last part involves lower limb deceleration just before foot strike; acts to slow forward propulsion in order to make contact with the ground)

the 5 component of stance phase

-initial contact (foot strike) -loading response (foot flat) -midstance (single leg stance) -terminal stance (begins at heel off) -preswing (ends at toe off)

Which one of the following positive likelihood ratios would classify a special test for a lateral ankle sprain as an "Excellent" clinical test?

24.52

the Ottawa rules. clinical judgement should prevail over the rules if the patient

-is intoxicated or uncooperative -has other distracting painful injuries -has diminished sensation in the legs -has gross swelling which prevents palpation of the malleolar bone tenderness

treatment of myositis ossificans "heterotypic ossification"

-it's prudent to avoid aggressive manual therapy interventions to the quadriceps or hamstrings that might irritate and cause rebreeding of the femoral periosteum -occasionally surgery to remove excessive bone, not worth the risk

characteristics common to all diarthrodial (synovial) joints

-joint cavity -ligamentous joint capsule -synovial membrane -synovial fluid (lubricant) -articular (hyaline) cartilage (on the ends of the long bones that form the joint)

growth and development factors that affect posture

-knees are slightly bowed (genu varum) until about 8 months -children then become knock kneed (genu valgum) until about age 3 -By age 6 leg alignment should naturally straighten

other factors that affect posture

-laxity of ligamentous structures (hypermobility) -muscle tonus -fascial and musculotendinous tightness

functions of the menisci

-load transmission (transmits 50-60% of weight bearing load in full extension, and approximately 85-90% in flexion) -shock absorption (protect the articular surfaces of the knee under loads that can reach 6x body weight) -joint lubrication (help drive the synovial fluid into the articular (hyaline) cartilage at ends of femur and tibia) -joint stability (deepen the articulating surface of the tibial plateau, particularly under values/varus and axial stresses)

distal tibial fracture time to radiographic union

-low energy fracture= 10-13 weeks -high energy fractures= 13-20 weeks -open fractures= 16-26 weeks (4-6months)

risk factors of achilles tendon ruptures

-males 30-50yr -beginner athlete engaging in an atypical athletic activity aka weekend warrior -oral or intratendinous injection of corticosteriods -hypercholesterolemia -gout -rheumatoid athritis -long term dialysis -renal transplant

to reach natural posture, have your patient

-march in place 10x -roll shoulders forward and backward 3x -nod head forward and backward 5x -inhale and exhale deeply

injuries to the posterior cruciate ligament

-much less common than ACL - MOI often fall on flexed knee, blow to anterior knee or posterior loading

when performing the medical screen on an orthopedic patient, clinicians should remember the following:

-musculoskeletal pain is affected by various joint movements -musculoskeletal pain is made worse/better bu various components of the medical screening process -lack of change in a patients symptoms should alert you to the potential presence of a medical related pathology

dynamic valgus alignment (position of no return)

-non contact situation -MOI: landing from a jump, cutting, sudden deceleration -extended hip and knee positions -valgus collapse -internal tibial rotation -pronanted foot

criteria for acute tibial compartment compression syndrome diagnosis and treatment

-normal = up to 10 mmHg -15-20= serial compartment pressure measurements -20-30= surgical consultation and admit -greater than 30= surgical fasciotomy

exertion tibial compartment compression syndrome involving dermatome and myotome

-numbness and/or tingling in the affecter dermatome -weakness or loss of motor function in the affected myotome

special test used to evaluate sending Larsen Johansson syndrome

-palpate the distal pole of patella for point tenderness -MMT of quadriceps

surgical treatment options for achilles tendon ruptures

-percuataneous -open (NONAUGMENTED; direct repair and 3 bundle technique) (AGUMENTED; gastrocnemius-soleus paratenon augmented rapier and platers tendon reconstruction)

clinical examination of posture with inexpensive clinical tools

-plumb bob (string and pendulum that hangs perpendicular to a surface) -inclinometers -goniometers -flexible rulers

lateral view examination of posture

-private area with a comfortable temp -males only in shorts, women in shorts and top to expose back, no shoes -the plumb bob should pass thru trigs of ear and acromion process

subtalar joint functions

-pronation (tibial internal rotation, talocrural plantar flexion, subtalar eversion and foot Abduction) -supination (tibial external rotation, talocrural dorsiflexion, subtalar inversion and foot Adduction)

anatomical movements of the lumbar spine

-sagital plane (flexion and extension) -frontal plane (right and left lateral flexion) -transverse plane (right and left rotation)

interpretation of radiographic images

A=alignment B=bone density C=cartilage space s=soft tissue

piriformis syndrome "false sciatica"

-spastic or tight piriformis compresses the sciatic nerve, -uncommon neuromuscular disorder -pain in the hip and buttock that radiates down the leg -often overlooked because it presents similar to lumbar radiculopathy or primary sacral dysfunction

4 primary mechanisms of achilles tendon ruptures

-sudden dorsiflexion of a plantar flexed foot -pushing off the weight bearing foot while extending the ipsilateral knee joint -sudden excess tension on an already taut tendon -a taut tendon struck by a blunt object

radiographic impinging in the diagnosis of syndesmosis sprain

-tibiofibular joint clear space in mortise and AP views is normally <5mm wide -tibiofibular overlap should be >10 (<10 implies a syndesmotic injury) -tibiofibular clear space >5 is abnormal, suggesting syndesmotic injury

Normal AROM for knee int rot at full extension (screw home mech) - exam 2

0 deg , bc the knee locks

KNEE EXTENSION ROM

0-10

KNEE FLEXION ROM

0-135 to 145

ANKLE EVERSION ROM

0-15

Ankle Dorsiflexion ROM

0-20

ANKLE INVERSION ROM

0-35

Ankle Plantar Flexion ROM

0-50

The statistic, negative likelihood ratio (-LR), is the ratio of a negative (-) test result in a people with the pathology to a negative test result in people without the pathology. Which one of the following values is the best -LR?

0.01

MMT scale

0/5: no movement 1/5: slight activation, no movement 2/5: active movement but unable to overcome gravity 3/5: active movement, gravity, but not against resistance 4/5: movement against some resistance 5/5: normal strength

Functions of the Menisci

1) In high load, 70% of compressive load is absorbed by menisci. (Lateral meniscus= most) 2) Wedge-shaped assist in lubrication of the joint. Acts as a space-felling mechanism, increasing synovial fluid delivery to the surfaces of tibia and femur. 3) 20% increase in joint friction following meniscal removal (Total meniscectomy= rare)

Functions of the menisci of the knee

1) Load transmission- Transmits 50-60% of weight baring load in full extension, and approx 85-90% in flextion 2) Schock Absorption- Protect the articular surfaces of the knee under loads that can reach 6X body weight 3) Joint lubrication- Help drive the synovial fluid into the articular (hyaline) cartilage at the ends of femur and tibia 4) Joint stability- Deppen the articulating surface of the tibial plateau, particularly under valgus/ varus and axial stresses

6 major static stabilizers of the tibiofemoral joint

1) MCL 2) LCL 3) ACL 4) PCL 5) Medial Meniscus 6) Lateral Meniscus

snapping hip syndrome (external coxa saltans) MOI

1) the post border of ITB or 2) ant border of glut max snaps over the Greater trochanter of femur

factors that contribute to a higher rate of non contact ACL injuries among women

1, decreased protective role of dynamic knee stabilizers 2. diminished ability to resist anterior tibial shear with muscle co activation 3. women typically have small ACLs 4. narrower femoral intercondylar notches than men 5. varying injury risks at different stages of menstural cycle

what surgical procedure is effective for a femoral cartilage injury w/o S/S of knee osteoarthritis

ACI and OAT would be effective

physical examination of the patellofemoral joint

1.walking gait 2.standing (knee joint and ankle/ subtler joints) 3.seated with knees at 90 degrees flexion (patella Alta and patella Baja, and extension of knee from flexed position J sign) 4.assessment of hip joint version (anteversion internal and retroversion external) 5.Supine evaluation of patella (translation in medial/lateral directions, lateral retinaculum, Q angle norms)

In the skeletally immature child, there are ___________ movable vertebrae in the lumbo-sacral spine?

10

during the first and last _____- of the stance phase, the body is supported by both lower extremities

10-12%

the perimeniscal capillary plexus penetrates

10-30% of outer medial meniscus and 10-25% of outer lateral meniscus

A typical computed tomography (CT) scan delivers the same amount of ionizing radiation to the patient as approximately ___________ standard X-rays?

100

Most people receive _____ sec to talk before being interrupted, try to give at least

11 sec, 2 minutes

patients with syndesmosis sprains will continue to improve for

12-18 months following the surgical procedure

HIP FLEXION GONI ROM

120 DEG

Q ANGLE FOR MEN

13

choosing wisely campaign is endorsed by _____ medical organizations representing more than ______- physicians, nurses, pathologist, radiologist and other health care professionals

16, 350,000

landing error scoring system, 3 kinds

17 errors are possible in 1 landing, LABORATORY VERSION using 3 trial averages, LESS REAL TIME, ILESS TEST

Q ANGLE FOR WOMEN

18

osteochondritis dissecans in femoral condyles present in about _______ of patients undergoing knee arthroscopic surgery

19%

Nearly a _______ ratio of size of the femoral head to the acetabulum creates a very stable joint

1:1

what is the ratio for the femoral head/acetabulum

1:1

grades of turf toe injury

1= microdamage of plantar complex 2 = partial tear of plantar complex 3 = complete tear of plantar complex

lThe hallux sesamoids play an important role in great toe biomechanics by dissipating weight-bearing and shear stress in the

1st MTP joint.

assessment of posture, inspected in ____ planes and ___ views

2 (sagital and frontal), 3 views (lateral, anterior, posterior)

during the course of each day, physical activity reduces the height and volume of intervertebral disc by about

20% (15-25mm most of which occurs in the first hour after rising)

there is a ___ increase in joint friction following meniscal removal

20%, "total meniscectomy" = rare

KNEE INTERNAL ROTATION ROM

20-30

more than ________ surgeries are performed each year in the USA for meniscal lesions

200,000

at 5 degrees of flexion the knee has

23 degrees of external rotation and 10 degrees of internal rotation

current estimates of recurrence of low back pain at 1 year range from ______. Most improvement of low back pain occurs within the first 6 weeks of onset

24-84%

Sinding-Larsen-Johansson Syndrom- exam 2

An irritation of the immature inferior pole of the patella.

ANKLE DORSIFLEXION ROM

20

approximately _ of the population has pes planus

20%

approximately __ of the population has pes cavus

20%

military basic training is directly compared to what

ACL injury rates bw men / women that has shown the greatest difference

the unhappy triad consists of an injury to

ACL, medial meniscus, and MCL

When assessing a patient for differences in true leg length, which of the following anatomical landmarks are used?

ASIS to distal medial malleolus

Structural/true leg length

ASIS to medial malleolus An actual difference in the length of the femur or the tibia of one leg compared to the othe

How many muscles attach to the sacrum, ilium, or both?

35

lateral (inversion) ankle sprain anatomy

ATF= primary restrain to inversion in plantar flexion CF= primary restraint to inversion in neutral and dorsiflexion PTF= strongest of the lateral ligaments

Good listening skills are essential for success in patient interviews and obtaining accurate medical histories from patients. We have discussed the tendency of physicians in the United States to interrupt patients while they were giving their opening statements. One study that was cited in both lecture and the Reiman (2016) text indicated that in order for patients to divulge their primary concern ("chief complaint"), they needed at least __________________ of uninterrupted speaking time to do so.

30 seconds

The stance phase accounts for about 60% of the gait cycle during walking, while the swing phase accounts for about 40%. According to the data from the classic study by Mann (1986) in the American Journal of Sports Medicine, what are the approximate proportions of the stance and swing phases during jogging, running and sprinting?

30% stance phase, 30% swing phase

KNEE EXTERNAL ROTATION ROM

30-40

achilles tendinitis is more likely to occur in those older than

35 years old

fibularis terminus is present in about

38-56% of population

the leg is divided into ___ osseofascial compartments by the interosseous membrane and __ inter muscular septa

4 (anterior, lateral, superficial posterior and deep posterior), 2

while low back pain affects nearly everyone at some time in their lives, in up to ___ of these cases,

85% no specific pathology is identified

The anterior cruciate ligament (ACL) provides approximately ________ of the resistance to anterior tibial translation, while the posterior cruciate ligament (PCL) provides approximately _______ of the resistance to posterior tibial translation.

85%; 90%

ankle fractures are one of the most common lower limb fractures and account for approximately

9% of all fractures

Forefoot pain is common and multifactorial with disorders of the 1st metatarsophalangeal joint (MTP) and hallux sesamoid complex (HSC) corresponding to

9% of foot and ankle injuries and 12% of great toe (hallux) injuries

the posterior cruciate ligament provides ___ of the resistance against posterior tibial translation

90%

Sallat reported that _______ of patients tear their medial patellofemoral ligament with a patellar dislocation

94%

Hip fracture from a fall is a major public health concern in the United States. Falls are responsible for ______ of the hip fractures in persons 65 years of age and older, and _________ of these patients will die within one year after fracturing their hip.

95%, 20%

Medial Synovial Plica Test

pos -- Pain or reproduction of friction symptoms imp -- Synovial plica syndrome

what percentage of hip fractures is due to a fall in the elderly? what percentage of ppl die every year due to the fall?

95%, 20%

sensitivity of Ottawa ankle rules

97.6 (96.4-98.9)

hyperkyphosis is a kyphotic angle _____ that is measured from a ______

>40 degrees (Cobb angle), lateral x-ray of T4-T12

Yellow flag signs

Abnormal signs and symptoms Vilateral symptoms Symptoms that are peripheralizing Progressive weaknessMultiple inflamed jointsPsychosocial stresses

Hyperkyphosis is defined as a kyphotic angle greater than ___________ degrees between T4 and T12 vertebral bodies that is obtained from a lateral x-ray using the Cobb method.

40

approximately _____ of cases have long term residual symptoms with as many as ____ of patients with acute ankle sprains developing chronic ankle instability

40-50%, 50%

HIP ABDUCTION GONI ROM

45 DEG

when the knee is in a position of flexion between 30-90 degrees, there are approximately

45 degrees of external rotation and 25 degrees of internal rotation

there is a ____ lifetime chance of developing knee pain

45%

our plantar flexors are ____ than our dorsiflexors

4x stronger

when testing myotomes, each isometric contraction must be held for greater than ___ seconds in order to allow for myotomal weakness to become evident

5

forefoot bones

5 metatarsals and 14 phalanges

lumbar abd sacral nerves

5 pairs of lumbar nerves, 5 pair of sacral nerves, 1 pair of coccygeal nerves

each lumbar intervertebral disc is about

5-10 mm tall

ANKLE PLANTAR FLEXION ROM

50

We now understand that genetics are responsible for as much as _______________ percent of all the cases of intervertebral disc degenerative disc disease (DDD), irrespective of the patient's sex.

50 to 75

osteochondritis dissecans of talus occurs in up to _____ of acute ankle fractures and sprains

50%

smith found the the medial patellofemoral ligament contributes to ___________ of the medial restraining force of the patella

53-67% (mean 60)

Nonverbal communication comprises _____ of all communication. the emotions context of care is highly related to nonverbal communication

55-97%

how many categories of possible hip pathologies?

58

the 5 lumbar disc contribute about _____- to the length of the lumbar spine

5cm (2in)

bunion is the lay term for a great toe exostosis or deformity while bunionette is the lay term for a

5th toe exostosis

Infants up until about 18 months of age will demonstrate slight genu varum, followed by genu valgum until about age 3. However, by the age of __________, most children's lower leg alignment will naturally straighten.

6 years

for mild injuries to the syndesmosis, conservative care produces an excellent long-term outcome, A return to sports within

6-8 weeks usually follows conservative care and a full recovery is expected in most cases

approximately _ of the population has pes rectus

60%

within each lamella of the annulus, the collagen fibers are arranged in parallel, but at

65 degrees to the sagittal plane

The fibular collateral ligament (FCL) provides approximately ________ of the resistance to adduction stress when the knee is in a position of about 25 degrees flexion.

70%

in high load situations, ____ of the compressive load is absorbed by the menisci, especially ny the _____

70%, lateral meniscus

During a patient interview, subject history alone can be used to determine the correct diagnosis with

74-90% accuracy

. Reiman (2016) cites evidence that suggests that the medical history portion of the patient interview, if well done, can identify the correct diagnosis in approximately ________ of patients.

75%

ACL percentage resistance to ant tib translation - exam 2

85%

During walking and running gait, approximately _______ of individuals make initial contact with the ground with a rear footstrike pattern.

85%

Which of the following statements are TRUE regarding the condition of a lumbar herniated nucleus pulposus (HNP)? (Read carefully and be certain to indicate all correct answers).

A lumbar herniated nucleus pulposus (HNP) is one of the most common diagnoses made by orthopedic spine specialists. Disc herniation-associated radiculopathy is now considered to be a biochemical as well as mechanical problem. Diagnostic imaging studies reveal that persons over the age of 60 have a high incidence of bulging and herni

Hyaline Articular cartilage (HAC)

A thin, dense covering of the ends of all bones that comprise the diarthorodial (freely movable) joints

most common locations for avulsion of tendons among adolescents

ASIS (sartorius origin), AIIS (rectus femoris origin), Ischial tuberosity (hamstring origin), greater trochanter of femur (gluteals insertion), lesser trochanter of femur (iliac and psoas insertion), pubis rams (adductors and pectineus origin)

lumbar lordosis

An excessive anterior curvature of the lumbar spine

Cardiovascular diagnosis

Acute Coronary Syndrome Congestive Heart failureDeep vein thrombosis (DVT) Peripheral arterial disease Abdominal Aortic Aneurysm

special test for scoliosis

Adams test (bend forward)

Blue flags

Also can affect prognosis because the client may feel unsupported by employer, which can negatively affect their motivation

2 Bundles of the ACL

At 0 degrees (full knee extension) -Posterolateral(PL) bundle is taut -Anteromedial (AM) bundle is loose At 90 degrees (Flexion) - Posterolateral (PL) bundle is loose - Anteromedial (AM) bindle is taut

Who created the 4 cardinal signs of inflammation

Aulus Celsius

Scheuermann's Disease

Inflammation of the vertebrae and cartilage occurs around the ring apophysis of the vertebral body Idiopathic bit its present in approx 10% of populationCommonly occurs between T10-L2

imaging technology that provides a comparable "real time view" to a physician

fluoroscopy and diagnostic ultrasound (images are not as clear)

Which ligament is the primary restraint to varus loads

LCL 55% at 5 degrees of flexion 69% at 25 degrees of flexion

posteromedial drawer test

pos -- increased external rotation of the medial tibial condyle imp -- pcl tear

Which of the following special orthopedic tests is/are appropriate for the diagnosis of an anterior cruciate ligament (ACL) tear? (Read this question carefully and be sure to indicate all the correct answers).

Lachman test anterior drawer test anterior drawer test with a KT-1000 arthrometer

anatomical movements that the clinician knows will be painful are done ___ to avoid an overflow of symptoms

Last

three general caused of knee swelling

BLOOD (hemarthrosis)= 1-2hr, feels doughy/ hard SYNOVIAL FLUID = slower 8-24hr, feels boggy on palpation, PURULENCE PUS= skin is red and hot

A 110-meter high sophomore hurdler on the high school track team comes to your clinic complaining of chronic pain in the popliteal fossa of his left leg. Which one of the following injuries/conditions ranks the highest on your differential diagnosis list?

Baker's cyst

femoral triangle

Base: Inguinal ligament Medial border: Adductor longus Lateral border: Sartorius contents: femoral vein, artery and nerve

Intra-abdominal diagnosis

Blunt intra-abdominal injury Appendicitis Acure Cholecystitis (Gall bladder) Hepatomegaly (liver) Splenomegaly (Spleen)

An 11-year old physically-active boy who comes to your clinic complaining of anterior hip pain must be evaluated to rule out which of the following injuries and/or conditions? (Please read this question carefully and identify all the correct answers).

Legg-Calvé-Perthes disease psoas muscle strain slipped capital femoral epiphysis

Choosing Wisely Campaign

CT scans are not necessary in the immediate evaluation of minor head injuries to children, PECARN should be used to determine if imaging is indicated

Neurological diagnosis

Cauda Equina Syndrome Meningococcal DiseaseMTBI/ post-concussion syndrome Primary brain tumor Parkinson's disease Myasthenia Gravis

The four cardinal signs of inflammation in Western medicine were first described by ___________________.

Celsus

Common Reg flags

Changes in bowel or bladder function Changes in muscle tone or ROM Changes in skin or skin lesions Chest pain with exertionIncreased urination with excessive thirst Unexplained weight loss (>10% in 10-21 days) Pain that has not improved with therapy

What is used to quantify the severity of both kyphosis and scoliosis

Cobb Angle

The Scoliosis research society condors the ___________________ measurement for scoliosis

Cobb angle as the gold standard

A patient's coping skills, previous treatment, and attitudes and beliefs are considered to be components of which "dimension of pain" as identified by Magee (2014)? (Read carefully and be certain to indicate all correct answers).

Cognitive

Clear communication between patient and clinician is critical for obtaining an accurate medical history. In order to practice culturally competent care in this process, clinicians should ________________________________.

Convey respect for adult patients by addressing them formally, e.g., "Mrs. Sanchez", "Mr. Kawamura". Use jargon-free terminology ("lay terminology") when talking with a patient. Target an eighth-grade level of comprehension when choosing your words when in conversation with a patient. If English is your primary language, determine the patient's level of fluency in the English language.

Which of the following statements accurately describe Legg-Calvé-Perthes disease? (Please read this question carefully and identify all the correct answers).

Legg-Calvé-Perthes disease occurs in boys 4 times more frequently than in girls.

The knee is a "mechanical compromise" between.....

Mobility and stability

Myotomes

Motor function

Which of the following surgical procedures for femoral articular cartilage injury have outcomes that are described as "restorative"? (Please read this question carefully and identify all the correct answers). Correct!

autogenous chondrocyte implantation osteochondral allografts osteochondral autografts

white on white zone

avascular inner 2/3 of the meniscus, nutrition comes from synovial fluid, poor healing potential

varus stress test at 0 deg and 20-30 deg

pos -- increased laxity/pain imp -- lcl sprain

valgus stress test at O deg and 20-30 deg

pos -- increased laxity/pain imp -- mcl sprain

Quadrant test

Pos - pain in SI joint/lumbar spine Ind - fracture, stenosis

Femoral Nerve Stretch Test

Pos - pain or neurological symptoms in the ant thigh/quad area Ind - nerve root impingement

thessaly test

pos -- joint discomfort and/or locking/catching imp -- lesion of medial or lateral meniscus

Straight Leg Raise test

Pos -- Lift unhealthy leg and no pain, then lower leg 10 deg, dorsiflex foot Ind--- nerve root impingement

Single Leg Standing test

Pos -- pain in SI joint/lumbar spine Ind - SI joint issues/lumbar pathology

clinical decision rules are used in the emergency department to

improve diagnosis or decreases unneeded clinical testing (Ottawa ankle/foot rules)

Which of the following statements accurately describe non-enzymatic glycation (NEG), a biochemical phenomenon associated with the ageing process. (Read carefully and be certain to indicate all correct answers).

NEG mostly affects human tissues with a low metabolic rate and a high content of collagen. NEG is a slow degenerative process that reduces the capacity of the affected tissue to absorb compressive loading and impacts.

thompson test

pos -- lack of movement of the foot imp -- Achilles tendon rupture

autografts with ACL reconstruction

central 1/3 of the patellar tendon, free looped double or quadrupled hamstring tendon

there are 31 paints of spinal nerve roots. regions:

cervical (C1-C8), thoracic (T1-T12), lumbar (L1-L5), sacral (S1-S5) and coccygeal (Co1)

Which one of the following is NOT a physical sign of injury?

Nausea

Functional/Apparent Leg Length Discrepancy

Navel to medial malleolus Leg-length difference that is attributed to something other than the length of the tibia or femur

Hip Scouring (Hip Quadrant) Test

Pos = Pain or symptoms in the hip are reproduced. Ind = A possible defect in the articular cartilage of the femur or acetabulum (e.g., osteochondral defects, arthritis), Labral tear

Tension (Bowstring) sign

Pos- - pain or tenderness Ind -- Sciatic nerve irritation or popliteal nerve impingement

each vertebral body consist of an outer shell of _______ that is reinforced internally by a vertical and horizontal arrays of ____________

cortical (compact) bone and trabecular (spongy) bone

motor= none and sensory= anterior and lateral thigh down the the knee

lateral cutaneous nerve L2, L3

is achieved by disc compression on one side and tension of the other

lateral flexion

results primarily from postmenopausal osteoporosis: older men with low testosterone can have this too

osteoporotic kyphosis

The screw home mechanism

Created by unequal sizes of the femoral condyles, and how the rotate ("spin") on the tibial plateau.

Patellar tap test (Ballotable patella)

pos -- a gradual rise of the patella imp -- intracapsular swelling/effusion in the knee joint

Reiman (2016) described nonverbal communication as a vital part of the patient interview, said to comprise 55% to 97% of all communication. Which of the following are examples of effective forms of nonverbal behaviors? (Read carefully and be certain to indicate all correct answers).

Forward leaning posture uncrossed arms and legs head nodding arm symmetry

Steves & Hootman (2004) believed that evidence-based medicine (EBM) was important to the practice of physical medicine. Which of the following statements accurately describe their perspective? (Read carefully, and select all the answers that you consider to be correct).

EBM promotes critical thinking within the professions of physical and occupational therapy. Having strong evidence that supports the efficacy of a treatment will facilitate third party reimbursement for physical therapists, occupational therapists, and other physical medicine providers. The application of EBM principles can improve the level of care delivered to our patients

external rotation recurvatum test

pos -- a marked difference in hyperextension imp -- PLRI instability

Which of the following statements accurately characterize femoroacetabular impingement syndrome (FAI)? (Please read this question carefully and identify all the correct answers).

FAI commonly involves results in damage to the acetabular labrum In its early stages, FAI manifests as a deep, continuous pain during prolonged sitting. FAI is most commonly found among adolescent males, ages 13-21. The anterior impingement test has high sensitivity (0.90) and is a good clinical test for examining a patient for FAI.

Motor and sensory innervation of the knee extensor muscles located in the thigh is provided by the __________________ nerve?

Femoral

Knee joint (tebiofemoral) unites the two longest levers in the body....

Femur and tibia

The tibial (medial) and fibular (lateral) sesamoids are located within the tendons of __ and form a portion of the plantar plate.

Flexor Hallucis Brevis (FHB)

Patellar apprehension test

pos -- apprehension or pt stops you imp -- laxity and/or predisposition to subluxations/dislocations

According to the Meyerding scale, an L4 vertebral body that has subluxed anteriorly 35% of the width of L5 vertebral body is classified as a ___________________ severity spondylolisthesis.

Grade 2

With regard to the manual muscle testing (MMT) grading system, a female patient who, when asked by you, is able to actively flex her arthritic right hip joint through the full (normal) range of motion is described as having grade _______ muscle strength?

Grade 3

dynamic stabilizers of the knee flexors

HAMSTRINGS (primary), biceps femoris, semitendinosus, semimembranosus, GASTROCENEMIUS (secondary)

Dynamic stabilizers of the knee Knee Flexors

Hamstrings (primary) - Biceps Femoris - Semitendinosus - Semimembranosus Gastrocnemius (Secondary)

Of all the component parts of the physical exalm, the _________________ is the most important

History

Cobb angle >40 degrees

Hyperkyphosis

memorizing lumbar plexus L1-L4

I, I Get Leftovers On Fridays

what injury is characterized by the lateral aspect of the knee has pain while running, tenderness, chronic, but full ROM

ITB friction syndrome

Major ligaments of the Hip joint

Iliofemoral ligament: Limits hip extension and hyperextension Pubofemoral ligament: Limits hip extension and abduction Ischiofemoral ligament: Limits hip internal rotation and extension

What muscle would be involved in an apophyseal avulsion of the lesser trochanter of the femur

Iliopsoas muscle group

In 2013, the American Academy of Pediatrics launched the "Choosing Wisely" campaign to inform health care providers about the dangers of the unnecessary use of computed tomography (CT) in making which of the following diagnoses, particularly in emergency department/emergency room care? (Read carefully and be certain to indicate all correct answers).

Immediate evaluation of minor head injuries and suspected concussions in children. Immediate evaluation of acute abdominal pain in children.

Osteitis pubis is an inflammatory condition involving the pubic symphysis that is typically caused by repetitive motion or training errors. Which of the following are known sources of the overloading that can produce this condition? (Read this question carefully and indicate all the correct answers).

Increasing the duration of an exercise program too rapidly. Running on concrete or asphalt roads or surfaces. Running on uneven terrain. Increasing the intensity of an exercise program too rapidly.

Negative likelihood ratio (-LR)

Indicates the likelihood that a negative test result was obtained in a person with the condition as compared to a person without the condition

Positive Likelihood Ratio (LR+)

Indicates the likelihood that a positive test result was obtained in a person with the condition as compared to a person without the condition

Dynamic stabilizers of the knee Rotator

Internal rotators - Pes Anserinus - Sartorius - Gracilis - Semitendinosus - Semimembranosus - Popliteus

5 characteristics common to all diarthrodial (synovial) joints....

Joint cavity ligamentous joint capsule Synovial membrane Synovial fluid (lubricant)Articular (Hyaline) cartilage (on the ends of the long bones that form the joint)

The results of a 2018 study by Van Der Straeten et al. used which of the following patient-reported outcome measures (PROs) in their short-term and medium term results of artificial meniscal implants? (Please read this question carefully and identify all the correct answers).

KOOS Lysholm Scale VAS for pain IKDC

surgical treatment for hallux valgus (bunions)

Keyhole Surgery, with 4 small (3 mm) incisions, inflicting much less iatrogenic damage.

an excessive posterior curvature of the thoracic spine (a slight one is normal and present in everyone)

Kyphosis

lumbar plexus

L1-L4

The spinal cord ends at the _______ vertebral level to become the cauda equina.

L2

Patellar tendon reflex

L3-L4

According to Reiman (2016), testing the patient's patellar tendon reflex, a component of the lower quarter screen, is actually a test of which one of the following spinal nerve roots?

L4

sacral plexus

L5-S2

treatment for turf toe injury

Long-term immobilization in walking boot or cast...Minimum of 4 to 6 weeks lost work/playing time with Grade 3 injury

an excessive anterior curvature of the lumbar spine, common in those with increased abdominal weight (pregnancy, pot bellies or beer bellies)

Lumbar Lordosis

proximal 1/3 of fibula with syndesmosis rupture, adults from external rotation force applied to ankle with foot either in supination or pronation

Maisonneuve fracture

Black flags

Mat affect clients prognosis due to limitations with insurance

The patient's subjective experience of pain can be made more objective using which of the following instruments? (Read carefully, and select all the answers that are correct).

McGill Pain Questionnaire Universal Pain Assessment Tool Numeric Pain Rating Scale (NPRS)

The patient's subjective experience of pain can be made more objective using which of the following instruments? (Read carefully and be certain to indicate all correct answers).

McGill Pain Questionnaire Visual analogue scale (VAS) Numeric pain rating scale (NPRS)

acute tibial compartment syndrome is a _________ that must be treated ____

Medical emergency, promptly to prevent irreversible muscle damage

Quadriceps angle (Q-angle)

Men= ~10 degrees or less Women= ~15 degrees or less

Starkey and Brown (2015) recommend that certain clinical findings warrant the termination of your orthopedic evaluation and immediate physician referral. Which of these findings are identified below? (Read carefully and be certain to indicate all correct answers).

Neurological testing indicates a motor and/or sensory loss. Active range of motion testing reveals a third degree muscle tear. Diminished posterior tibial arterial pulse Ligamentous testing ("special tissue tests") reveals gross joint instability. An absent or diminished patellar tendon reflex.

Eversion Stress Test (Talar Tilt)

pos -- excessive laxity or pain imp -- deltoid ligament sprain

Ultimate goal of the orthopedic exam is?

Obtaining a clinical or working diagnosis

Screw- home mechanism

Open Kinematic Chain - In non-weight bearing situations, the tibia externally rotates on the fixed femur Closed Kinematic Chain - In weight- bearing situations, the femur internally rotates on the fixed tibia

__________________________________ is an adolescent condition that involves the inflammation of the patellar tendon at its insertion onto the tibial tubercle.

Osgood-Schlatter's disease

Strongest ligament in the knee

PCL Provides 90% resistance to posterior tibial translation

LACHMAN TEST

POS -- INCREASED AMT OF ANT TIB translation IMP -- acl sprain

Ely's Test

POS = The hip on the side being tested flexes, causing it to rise from the table IND = rectus femoris tightness

Thomas Test

POS = The involved leg rises off the table iND = Tightness of the iliopsoas muscle group

Trendelenburg Sign/Test

POS = The pelvis lowers on the non-weight-bearing side IND = gluteus medius weakness

Anterior Impingement Test

POS = hip pain IND = Femoral acetabular impingement

Ober's Test

POS = leg is unable to adduct to horizontal. IND = Tightness of the IT band, predisposition to IT band friction syndrome

treatment for transverse and longitudinal arch sprain

PRICE, padding, strengthen muscles and stretch achilles tendon

observation and palpation for posture, posterior aspect

PSIS position, spinal alignment, scapular positions

Inversion Stress Test (Talar Tilt)

pos -- excessive laxity or pain imp -- lateral ankle sprain of the CFL

how to practice evidence based medicine: identify and define clinically relevant questions using PICOT

Patient populations, Intervention of interest, Comparison intervention or issue of interest, Outcome of interest, Time of interest

Forefoot pain (and ulceration) is most commonly found among patients who present in clinic with:

Pes cavus arches, Hammer toes, Equinus (limited dorsiflexion), Diabetes

six dimensions of pain

Physiological Sensory Affective Cognitive Behavioral Sociocultural

Hoover test

Pos - if you don't feel effort , aka lack of effort Ind -- malingering

Brudzinski-Kernig test

Pos - neurological pain at a, reduction of pain at b, no pain at c Ind - meningitis

ACL has two bundles

Posterolateral bundle and anteromedial bundle

Well Straight Leg Raising Test

Pos—lift healthy leg and leg on table has pain Ind - nerve root impingement or herniated disk

an eponym associated with several fractures involving the ankle joint. First degree (a single malleolus is fractured) Second degree (both malleoli are fractured) Third degree (trimalleolar fracture involving the articular surface of the tibia)

Potts fracture

The iliopsoas group:

Psoas (major, orgin= T12-L5 vertebral bodies, insertion= lesser trochanter of femur) Iliacus (orgin= iliac fossa, insertion= lesser trochanter of femur) Psoas Minor

Prolonged pronation of the subtalar joint does not give the foot time to re-supinate and become rigid lever at toe off. There is no absolute degree of pronation that is considered over -pronations unlike

Q angle

dynamic stabilizers of knee extensors

QUADRICEPS FEMORIS, rectus femoris, vastus lateralis, vastus intermedius, vastus medialis

patellofemoral joint dynamic restrains

QUADRICEPS FEMORIS, recutus femoris, vastus medialis, vastus lateraalis, and vastus intermedius

Dynamic stabilizers of the knee Knee extensors

Quadriceps Femoris - Rectus Femoris ( Acts at both hip and knee) -Vastus Lateralis -Vastus Intermedius - Vastus Medialis

Which of the following statements reflect current clinical management strategies for the posterior cruciate ligament (PCL)? (Please read this question carefully and identify all the correct answers).

Quite commonly, clinicians recommend a "wait and see" (nonsurgical) approach to patients who have moderate to severe PCL sprains.

Ober's Test

pos -- femur adducts to horizontal or unable imp -- IT band tightness

Zones of meniscal Vascularity

Red-on red zone - Vascularized outer 1/3rd of the meniscus - BEST healing potential Red-on- white Zone - Transitional Zone 3mm-5 from meniscosynovial junction - Intermediate healing potential White-on- white Zone - Avascular inner 2/3rd of the meniscus - Nutrition comes from synovial fluid - Poor healing potential

The functional characteristics of the menisci at the tibiofemoral joint include which of the following? (Please read this question carefully and identify all the correct answers).

Reducing joint pressure by increasing the contact area between the femur and tibia Increasing tibiofemoral joint lubrication through movement of the synovial fluid. Improving joint stability due to their concave shapes, helping to keep the "ball" (femoral condyles) on the "table" (tibial plateau).

4 cardinal signs of inflammation

Rubor (redness) Tumor (swelling) Calor (heat) Dolor (pain)

Who created the 5th cardinal sign of inflammation?

Rudolf Virchow (he also created the cell theory)

Achilles Tendon reflex

S1-S2

What bones make up the SIJ

Sacrum (provides stability and transmits body weight from the spine to the pelvic region) and two innominate

Pediatric pelvic avulsion fractures are essentially ____________________ fractures that commonly occur at the ASIS, AIIS or ischial tuberosity apophyses.

Salter-Harris Type I

Pediatric pelvic avulsion fractures are essentially __________________________________ fractures that commonly occur at the ASIS, AIIS, or ischial tuberosity apophyses.

Salter-Harris Type I

inflammation of the vertebrae and cartilage occurs around the ring apophysis of the vertebral body, leads to anterior wedging of the vertebra

Scheuermanns Disease

Red flags

Signs and symptoms suggestive of serious pathology and very poor prognosis in orthopedic clients

Which of the following statements are true regarding providing culturally competent care for your patients? (Read carefully and be certain to indicate all correct answers).

The clinician should be alert and look for nonverbal cues that his/her patient is anxious and/or uncomfortable with the way that medical care is being provided. When performing the orthopedic examination of a patient, the clinician must respect the patient's religious beliefs and not include or perform techniques that will offend her/him. The clinician should respect the patient's perspective(s) on complementary and/or alternative medical approaches.

When evaluating a suspected knee joint injury, an important consideration for a meniscal injury being present is the fact that _______________________________________________. (Please read this question carefully and identify all the correct answers).

The medial meniscus has an attachment to the deep medial collateral ligament (joint capsule) and is commonly injured when valgus stress is the mechanism of injury (MOI).

screw home mechanism

created by unequal sizes of the femoral condyles and how the rotate ("SPIN") on the tibial plateau, humans do not spend enough time in full extension to utilize this

Somatic Pain

Soma= bodyPain originates in muscle, soft tissue and skinSuperficial/ deep

sacral plexus memory

Some Idiots Say Pretty Please

Which of the following statistical values is/are helpful for the clinician to know when choosing a special orthopedic test in the evaluation of a lower extremity injury? (Read carefully, and select all the answers that you consider to be correct).

Specificity Accuracy Reliability Answer validity Answer sensitivity

International Classification of Functioning, disability and health (ICF Model)

The World Health Organizations broader framework for the exam process, focuses on the need to consider the entire pt in the grand scheme of things

Steves & Hootman (2004) believed that evidence-based medicine (EBM) was important to the practice of physical medicine. Which of the following statements accurately describe their perspective? (Read carefully and be certain to indicate all correct answers).

The application of EBM principles can improve the level of care delivered to patients Having strong evidence that supports the efficacy of a treatment will facilitate third party reimbursement for physical therapists, occupational therapists, and other physical medicine providers. EBM promotes critical thinking within the professions of physical and occupational therapy.

Which of the following are considered to be anatomical and/or biomechanical functions of the menisci at the tibiofemoral joint? (Read this question carefully and be sure to indicate all the correct answers).

The menisci aid joint lubrication by driving the synovial fluid into the avascular hyaline articular cartilage on the ends of the femur and tibia. The menisci deepen the articulation between the femur and the tibia, thus improving joint stability The menisci serve as effective shock absorbers, distributing compressive loads that can be as high as six times body weight and more.

Which of the following statements are correct regarding the clinician's utilization the patient's contralateral (normal) limb when conducting your orthopedic examination?

The patient's contralateral limb serves as a reference for what is normal for that patient The patient's contralateral ("normal") limb is used to determine whether bilateral symmetry exists for sensory and motor functions. both A and B The patient's contralateral ("healthy") limb should be used as a comparator when observing for the cardinal signs of inflammation ALL THE ABOVE

yellow flags

Their presence likely suggest poor prognosis, but not serious pathology

conservative treatment for hallux valgus (bunions)

This "night splint" uses the biomechanical principle of creep deformation to eventually realign the great toe

Largest diarthrodial joint in body

Tibiofemoral Joint (knee)

Which of the following are basic competencies that Reiman (2016) says are expected of physical medicine clinicians regarding diagnostic imaging? (Read carefully and be certain to indicate all correct answers).

Understanding the radiographic report from the referring radiologist as well as the terminology used to describe the findings. Being able to explain imaging findings to patients and discuss diagnostic imaging with other professionals. Being able to recognize precautions and contraindications to treatment and modify the plan of care accordingly, based on the results of the imaging report.

Which of the following principles apply to the physical examination portion of the orthopedic assessment of a patient? (Read carefully and be certain to indicate all correct answers).

Unless bilateral movement is required, the normal (uninjured) side is tested first. Answer When evaluating the integrity of ligaments, stress is applied gently and repeated several times; the stress is gradually increased (up to the point of pain) in order that maximum instability can be demonstrated without causing muscle spasm. When individual myotomes are being tested with manual muscle testing, the patient should hold each isometric contraction for at least 5 seconds to determine if any myotomal weakness becomes evident.

Which of the following statements correctly describe the medical history, physical signs, and symptoms associated with Sinding-Larsen-Johansson syndrome? (Please read this question carefully and identify all the correct answers).

Upon palpation, the distal pole of the patella will be point tender. The patient will typically demonstrate a weakness in manual muscle testing of the knee extensor muscles in the involved limb. The patient is typically a pre-adolescent boy or girl who participates in sports and physical activities that involve running and jumping.

Clear communication between patient and clinician is critical for obtaining an accurate medical history. In order to practice culturally competent care in this process, clinicians should ________________________________.

Use jargon-free terminology ("lay terminology") when talking with the patient. Determine the patient's level of fluency in the English language both A and B Convey respect for adult patients by addressing them formally, e.g., "Mrs. Garcia". ALL THE ABOVE

MCL injury MOI

Valgus loading of weight bearing knee

Regarding palpation of a patient, the clinician should take particular note of which of the following? (Read carefully, and select all the answers that are correct).

Varying tissue densities in the quadriceps femoris The presence of one or more trigger points in the posterior calf Crepitus near a suspected fibular fracture Increased skin tissue temperature when compared to the contralateral normal ankle

Visceral pain

Viscera= gutPain originates in internal organsAching/squeezing pain

Triage originated in _____ and is now used by

WW1, EMS and ER personnel at or following mass casualties

surgical treatment of lateral (inversion) ankle sprain

Watson-jones, brostrom Evans or christen snook. All 3 use fibularis brevis

Factors increasing the Q-angle

Width of hips Genu Valgum Poorly positioned tibial tubercle Tibial rotation ('tibial torsion") Pes Planus ('Flat Feet")

radiography discovered by _____ allows

Wilhelm Röntgen, internal structures of the body to be viewed with non-invasive techniques

acetabular labral tear symptoms

a locking, clicking or catching sensation in the hip joint, pain in hip or groin, stiffness or limited range of motion in the hip. Many tears have NO signs or symptoms

patellar instability chronic etiology

a multi factorial problem, lateral tibial tubercle, increased Q angle (valgus), patella alta, quadriceps imbalance

The specificity of a clinical or diagnostic test is defined as the percentage of patients with ___________________________________________

a positive diagnostic test result who have the suspected disease or injury.

SCFE MOI

a shear load

what is the cause of the greek (Mortons) foot type?

a shortened 1st metatarsal and an elongated 2nd metatarsal

In the human body, fibrocartilage is __________________________________________. (Read very carefully, as only one (1) answer is correct)

a specialized type of cartilage found only in some synovial/diathrodial joints

exertion tibial compartment compression syndrome: pain and disability occur after

a specific, predictable amount of exercise

validity

a test is valid if it measures what it claims to measure

hyaline articular cartilage

a thin, dense covering of the ends of all bones that comprise diarthrodial (freely moveable) joint, water (65-80%), collagen (10-20%), proteoglycan (10-15%), chondrocytes (5%)

pronation-abduction

a transverse medial malleolus fracture, followed by a rupture of the anterior and posterior tibiofibular ligaments and the interosseous ligament and than an oblique fibular fracture

keystone of lateral longitudinal arch

cuboid

brush/stroke/bulge test

pos -- fluid seen or felt when brushing imp -- extracapsular swelling/effusion

common yellow flags

abnormal signs and symptoms, bilateral symptoms, symptoms that are peripheralizing, progressive weakness, multiple inflamed joints, psychosocial stresses

distal tibial fractures may be more prone to nonunion than proximal fracture due to

absence of muscular soft tissue envelope and the inherent zone of hypovascularity

formed by the triceps surae muscle group (gastrocnemius and soleus). Frequent and repeated loading predisposes it to overuse pathologies

achilles tendon

order of ROM during orthopedic examination

active - active assisted - passive- resisted

Which of the following are modifiable risk factors for knee osteoarthritis, a condition that affects more than one-third of the 65 and older population in the United States? (Please read this question carefully and identify all the correct answers).

activity level body weight

exacerbations of previous low back pain that act like the first onset

acute on chronic pain

achilles tendinitis is of ____ and pain is prevalent during ______

acute onset, push off activities

initial onset of pain, for 90% of patients the symptoms resolve spontaneously in 2-4 weeks

acute pain

The medial border of the femoral triangle is comprised of the _____________________________.

adductor longus

femoral triangle medial border

adductor longus

Primary hip internal rotators

adductor longus, adductor Magnus, adductor brevis, gluteus medius, gluteus minimus, tensor fasciae latae, pectine, and gracilis

Primary hip adductor muscles

adductor longus, adductor Magnus, adductor brevis, gracilis, pectinous, biceps femoris (long head only), gluteus Maximus (posterior fibers), and obturator externus

periosteum of the tibia contains what

afferent free afferent free nerve endings and causes pain in pt when injured or structurally damaged, periosteum can have bone bruise, stress fracture

risk factors for legg calve perthes disease

age (4-10), sex (4:1 boys:girls), race (white), and genetic mutations

the hamstrings are _______ of the ACL and ____________- anterior tibial translation

agonists, help resist

case study -- gen ant hip pain in young boy could indicate which of the following

all of the above -- iliopsoas strain / scfe / legg-calves perthe

the tibial nerve innervates

all of the muscles of the posterior leg and those on the plantar aspect of the foot (15 muscles)

two vertebral bodies form an

ampiarthrodial joint

patients with talar AVN frequently present with progressive

ankle pain and limited range of motion

idiopathic, risk factors include males, late adolescence/ early adulthood and heredity (HLA-B27 gene)

ankylosing spondylitis

20-30 sheets of collagen tightly packed together in a circumferential pattern on the periphery of the disc

annulus fibrosus

consists mainly of type 1 collage on the periphery and type 11 collage centrally

annulus fibrosus

the quadriceps are __________- to the ACL and __________- anterior tibial translation

antagonist, increase

syndesmosis (high ankle) sprain anatomy

anterioinferior tibiofibular ligament, posterioinferior tibiofibular ligament, transverse tibiofibular ligament and interosseous membrane

3 muscle compartments of the thigh: _____________ hip flexors and knee extensors, _________ hip adductors and knee internal rotators, _______ hip extensors and knee flexors

anterior (femoral nerve), medial (obturator nerve), posterior (sciatic nerve)

tibial compartment compression syndromes

anterior (most common location), deep posterior, lateral, and superficial posterior (very rare)

major ligaments of the motion segment

anterior = anterior and posterior longitudinal ligaments posterior= ligamentum flavum, supraspinous ligament and capsular ligaments of the facet joints

Which of the following contribution to the static stability of the tibiofemoral joint? (Read this question carefully and be sure to indicate all the correct answers).

anterior and posterior cruciate ligaments medial and lateral collateral ligaments negative intraarticular (joint) pressure

Which of the following special tests are designed to evaluate the status of the anterior cruciate ligament (ACL)? (Please read this question carefully and identify all the correct answers).

anterior drawer test Lachman test

the thinnest in the lumbar region of the spine and limits extension

anterior longitudinal ligament

a motion segment has two elements or portions

anterior portion = amphiarthrodial joint and posterior portion = diarthrodial joint (4)

supporting ligaments of the proximal tibiofibular joint

anterior proximal tibiofibular ligament, posterior proximal tibiofibular ligament and interosseous membrane

lateral ankle ligaments

anterior talofibular ligaments, calcaneofibular ligament and posterior talofibular ligament

medial ankle ligaments (deltoid ligament)

anterior tibiotalar ligament, tibiocalcaneal ligament, tibionavicular ligament and posterior tibiotalar ligament

Osteoporotic kyphosis: because of the osteoporosis,

anterior wedge fractures occur to multiple vertebrae in the middle to upper thoracic spine and contributes to stooping and a decrease in height

Scheuermann's disease is an idiopathic condition that affects approximately 10% of the general population. This inflammatory, degenerative condition leads to _________________ wedging of multiple vertebrae, and creates an exaggerated __________________ curvature of the spine.

anterior, posterior

With sacroiliac nutation, the superior aspect of the sacrum moves ____________________ and stress is put on the _______________________ ligament.

anteriorly; sacrotuberous

PCL Bundles (2)

anterolateral bundle is taut in flexion posteromedial bundle is taut in extension

Physical sign

any objective evidence that is readily apparent to observers and are measurable or quantifiable

Symptoms

any subjective evidence, any change of the body perceived by the patient, described by the patient

spina bifida can happen_____ when the neural tube does not close all the way, the vertebra that protects the spinal cord does not form and close as it should

anywhere along the spine if the neural tube does not close all the way

intra-abdominal differential diagnosis and key signs

appendicits, pain that localizes at mcBurneys point, rebound tenderness in the lower R quadrants (blumbergs sign)

patellofemoral joint structure and function

articular cartilage on the posterior surface of the patella effectively distributes compressive forces, the patella protects the tibiofemoral joint anteriorly, the patella increases the mechanical advantage of the quadriceps by 30-45% depending on knee joint angle

Apophyseal avulsion signs and symptoms

audible pop or snap at the time of injury, pain and hyperesthesia on the outer femoral surface

What type of joint in the hip?

ball and socket

transverse arch of the midfoot is formed by the

bases of the 5 metatarsals, 3 cuneiforms and the cuboid

evidence based practice incorporates 3 elements of the decision making process of patient care

best available research, clinical expertise and circumstances and values of the individual patient

supra patellar bursa

between femur and quadriceps tendon, communicates with synovial membrane of knee joint

prepatellar bursa

between patella and skin

popliteal bursa

between popliteus tendon and capsules communicates with synovial membrane of knee joint

deep infra patellar bursa

between tibia and ligamentous patella

subcutaneous infrapatellar bursa

between tibial tuberosity and skin

An apophyseal avulsion of the ischial tuberosity involves which one of the following muscles?

biceps femoris

Which of the following muscles act concentrically to cause external rotation at the knee? (Please read this question carefully and identify all the correct answers).

biceps femoris

Primary hip extensor muscles

biceps femoris (long head only), semimembranosus, semitendinosus, gluteus Maximus, gluteus medium (secondary) and adductor Magnus (secondary)

sciatic nerve arises from the L4, L5, S1, S2, and S3 nerve roots, almost immediately (at the branch level) the sciatic nerve

bifurcate into a tibial portion and a fibular portion

dual energy x-ray absorptiometry (DEXA) produces very small doses of ionizing radiation and enables quantification of

bone mineral content, bone mineral density, lean body mass, and body fat %

3. In Chapter 6, Reiman (2016) suggested that the main function of plain film radiography ("x-rays") is to rule out fractures and conditions such as ______________________________. (Read carefully, and select all the answers that you consider to be correct).

bone tumors hip osteoarthritis

patellofemoral joint static stabilizers

bony architecture of the patella, trochlear groove of the femur, medial patella femoral ligament (primary restraint against lateral patellar translation (about 60%), medial patellomeniscal ligament (13%), lateral retinaculum (10%), and lateral patellofemoral ligament

Which of the following diagnostic imaging techniques expose the patient to ionizing radiation?

both CT and fluoroscopy

while about 300,000 lumbar discectomies are performed annually in the USA, controversy still exists regarding the most appropriate treatment methods for herniated nucleus pulposus, disc herniation associated radiculopathy is now thought to be

both a biochemical and a mechanical disorder

genu varum vs genu valgum vs genu recurvatum

bowed legs vs knock knees vs back knee

T1 weighted appearances

bright: adipose tissue and bone marrow dark: fluid, fibrocartilage, ligaments and tendons

T2 weighted appearances

bright: fluid, infection, tumor, inflammation dark: fibrocartilage, ligaments, tendons, muscles, bone marrow

slocum drawer test ALRI

pos -- increased amt of ant tib translation imp -- anterolateral instability

during the stance phase, increased dorsiflexion and knee flexion occur on the affected side, resulting in a decreased step length

calcaneal gait, caused by paralysis or weakness of the plantar flexors or painful weight bearing on the forefoot or toes

the largest and strongest tendon in the body

calcaneal tendon (achilles tendon)

in the first years of life, the junction between the ring apophysis and the rest of the vertebral body is made through a __________ but becomes

cartilage layer, completely ossified around 18 yrs old

a bundle of spinal nerves and nerve rootlets consisting of the 2nd - 5th lumbar nerve prayers, the 1st -5th sacral nerve pairs and the coccygeal nerve

cauda equina (horses tail)

what the clinician sees during mid stance

center of gravity and line of gravity are directly over the foot which is flat on the ground

two secondary anterior curves

cervical= development begins about 3 months as the infant lifts the head the explore the world, normal lordotic curvature develops as a result lumbar= begins 6-8 months as the child being to crawl, sit, and eventually walk, normal lordotic (anterior) curvature is created

common red flags

changes in bladder/ bowel function, changing in muscle tone or ROM, changes in skin/ skin lesions, chest pain w exertion, increased urination w excessive third, unexplained weight loss (over 10% in 10-21 days), pain that has not improved w therapy, constant headache/ dizziness, shortness of breath, no discernible pattern of symptoms, hx of cancer, presence of unusual vital signs, unusual periods, rebound tenderness

fick angle = the normal toe out position that we all have in stance

children= 5 degrees and adults = 12-18 degrees

In young children, direct trauma to greater trochanter from a fall will likely cause ______, in elders with osteoporosis, a lateral fall on the hip will typically ___

chondral damage, fracture the proximal femur

achilles tendinosis is _____ in nature and may or may not have clinical symptoms with activity. often does not respond to anti inflammatories

chronic

defined as low back pain that does not resolve within 3 months or low back pain that partially improves but never completely goes away

chronic pain

lateral meniscus anatomy

circular (O), covers more of the tibia, uniform size, more mobile

plantar fasciitis pain may be more severe is

climbing stairs or performing intense activity (running hills/sand dunes)

A single position in which maximum bone/ joint surface area contact occurs. Attachments of the ligaments are farthest apart and under tension. Capsular structures are taut. The joint is mechanically compressed and difficult to distracted

closed packed position

The Patient Specific Talus Spacer implant, made from ___ is modeled from CT imaging to fit each patient's anatomy.

cobalt chromium alloy

PCL injury treatment plan - exam 2

commonly wait and see, nonsurgical treatment for mod-severe pain

syndesmosis sprain MOI

compressive or torsional loads

criterion related validity (relationship to other measures?)

concurrent validity= does the test relate to an existing similar measure? predictive validity= does the test predict later performance on a related criterion

According to Reiman (2016), which of the following represent cardiovascular conditions that the physical medicine clinician must be watchful for being present in her/his patient while performing the medical screening examination? (Read carefully and be certain to indicate all correct answers).

congestive heart failure deep vein thrombosis peripheral arterial disease abdominal aortic aneurysm acute coronary syndrome

apophyseal avulsion clinical treatment plan options

conservative: rest, protected weight bearing, read surgical: not the primary option, DRIF got avulsed bone with >3cm gap

sports hernia treatment

conservative= rest, strengthen affected muscles surgical=only after 9-12 weeks of conservative management fails

avascular necrosis also known as osteonecrosis

death of bone tissue due to a lack of blood supply, can lead to tiny breaks in the bone and the bones eventual collapse, most common in people aged 30-50

Femoroacetabular Impingement (FAI) is characterized by

decreased clearance and abnormal contact between the femoral head-neck junctions and the acetabular rim. 3 types: pincer impingement, cam impingement, and combined

sex hormones may negatively affect the physiological and mechanical properties ( decreased tensile strength) of the ACL, specifically causing:

decreased fibroblast proliferation, decreased pro collagen synthesis, and decreased in vitro tensile strength of ACL

anterior tibial compartment nerve and artery

deep fibular nerve and anterior tibial artery

semimembranosus bursa

deep to the semimembranosus tendon

baseline measurement of vital signs is necessary for

determining changes over time and the identification of a potential disease

Characterized by progressive loss of nerve fibers that predisposes the patient to: numbness, tingling, burning sensations, inability to feel pain or temp changes, increased sensitivity to touch, muscle weakness, ulcerations

diabetic neuropathy

The posterior portion of the vertebral motion segment is classified as a/an __________________________ type of joint.

diarthrosis

Rather than producing movement at the SIJ, the muscles around the pelvis are more likely involved

directly or indirectly in helping to provide stability to the joint

conditions that warrant termination of the orthopedic evaluation during palpation and ROM test

disruption of the contour of bone, malalignment of joint structures, complete grade 3 muscle tears (surgery is needed with 72hr!!)

supporting ligaments of the distal tibiofibular joint

distal anterior tibiofibular ligament, distal posterior tibiofibular ligament and interosseous membrane

considered as a sydensmosis joint

distal tibiofibular joint

talocrural joint

dorsiflexion 0-20degrees and plantar flexion 0-50 degrees

functions of anterior tibial compartments muscles

dorsiflexion of talocrural joint and extension of toes

pronation 15-30 is a combination of

dorsiflexion, hindfoot eversion and forefoot abduction

Legg-Calve-Perthes Disease pathological process in 4 phases

early phase of necrosis, revascularization with bone deposition and resorption, bone healing, and residual deformity

sports hernia etiology

eccentric loading of abdominal muscles, non palpable, small direct and indirect hernias

somatotypes

ectomorph (skinny), mesomorph (normal), endomorph (fat)

slocum drawer test AMRI

pos -- increased amt of ant tib translation imp -- anteromedial instability

fluoroscopy pros and cons

emit low doses of ionizing radiation but are generally lower quality images than x-rays

grade 3 sprain

empty end feel, complete rupture/ tear, greater than 10mm more laxity than normal joint

atypical scfe causes may include

endocrine disorders, metabolic disorders, radiation therapy

what the clinician sees during preswing

ends when the last toes comes off the ground

in a normal, healthy disc the nucleus distributes the load

equally through the annulus

lumbar musculature

erector spinae (spinalis, longissimus, illiocostalis) intermediate layer (multifidus) deep muscles (semispinalis, orators, interspinales and intertransversarii)

menstural cycle study used

estrogen and progesterone measured via saliva, and KT 2000 arthometer to quantify ACL laxity and stiffness

reproductive hormones involved in ACL - exam 2

estrogen(estraidol), testosterone, relaxin, progesterone

receptors for what are all present in the ACL

estrogen, progesterone, relaxin and testosterone

CT scans are not necessary in the routine __________

evaluation of abdominal pain

deltoid ligament limits excessive

eversion

frontal plane movement where the lateral border of the foot moves away from the midline

eversion

medial ankle sprain (infrequent) MOI

eversion

functions of the lateral tibial compartment muscles

eversion and plantar flexion

Slipped Capital Femoral Epiphysis (SCFE) risk factors

excessive weight or obesity (most patients are about the 95th percentile), family history of SCFE, endocrine or metabolic disorder

prevention of ACL injury

exciting research shows a 39-73% reduction in the ACL injury rates when performing a preventative training program

randomized controlled studies, quasi experimental studies

experiments studies are single studies Level 2 B

is achieved by posterior compression and anterior tension

extension

The iliofemoral ligament at the hip limits which of the following anatomical movements at the hip? (Please read this question carefully and identify all the correct answers).

extension and hyperextension

extension of toes anterior tibial compartment muscle degrees

extensor digitorum longus= 1 degree extensor hallucis longus = 1 degree

Meniscus is. a dense ______________ compose of primarily ______ and ____________

extracellular matrix, water, type 1 collagen

content related validity (appropriate content?)

face validity= does the test appear to measure what it aims to measure construct validity= does the teat relate to an underlying theoretical concept

also know a spondylosis or degenerative joint diseases, know the age of your patients. Pathology= a localized breakdown of one or more of they synovial joints of the spine

facet joint osteoarthritis

An accentuated lumbar lordotic curvature, often associated with Janda's "lower crossed syndrome", is commonly due to _______________________________________.

facilitated iliopsoas muscles and inhibited abdominal muscles

risk factors of lumbar disc herniations

family history and genetics, occupational actives, obesity

anatomical (intrinsic) risk factors for ACL injury

female sex, size offemorla intercondylar notch, tibiae with posterior slope, ACL volume, ligamentous laxity, lower leg alignment, genetic predisposition previous ACL injury

The sensorimotor innervation to the rectus femoris muscle is provided by which one of the following peripheral nerves

femoral

motor= quadriceps femoris, iliacus, pectine and sartorial and sensory= anterior thigh and medial leg

femoral nerve L2, L3, L4

femoral triangle contents

femoral vein, femoral artery, femoral nerve (VAN)

The screw-home mechanism increases the stability at the tibiofemoral joint at terminal extension. To achieve this in closed kinetic chain (CKC) mode, the _________________ __________________ rotates on the fixed ________________________

femur, internally, tibia

3 types of human cartilage and common characteristics

fibrocartilage, hyaline articular cartilage, and elastic. all mostly avascular, aneural, and alymphatic

The tendons of the ADductor Hallucis and ABductor Hallucis muscles have

fibrous insertions into the tibial and fibular sesamoidsrespectively.

lateral tibial compartment muscles

fibularis longus and fibularis brevis

grade 1 sprain

firm end point, no instability, no increase laxity; 0-5mm more laxity than the patients normal joint

diagnosing osteitis pubis

flamingo view x-ray, two stress x ray views of the symphysis pubis, mild cases may not be evident with this image

is achieved by each intervertebral disc being slight compressed anteriorly and resisted by tension in the posterior annulus fibrosus

flexion

MOI for herniated disc

flexion ->nucleus pulposus pressure against the posterior annulus fibrosus

movement at the amphiarthroses of the lumbar spine

flexion and extension

anterior drawer test

pos -- increased amt of ant tib translation imp -- acl sprain

weight distribution in the foot

heel = 50-60% metatarsal heads= 40-50%

risk factors for hallux valgus

heredity, rheumatoid arthritis, wearing tight, narrow shoes

in both acute and chronic compartment syndromes, __________________________; without treatment, muscle and nerve cells can die

high intracompartmental pressure impairs blood flow and causes excessive pain

achilles tendinitis is more likely in

high milage runners and ballet dancers, and conversely in sedentary and obese persons

pubofemoral ligament limits what motions

hip abduction / hyperextension

piriformis allows for what movement

hip abduction, ext rot

obturator nerve conducts what movement

hip adduction and knee internal rot

The _________________________ muscle group is located in the medial compartment of the thigh and is innervated by the _________________________ nerve

hip adductors, obturator

The _________________________ muscle group is located in the medial compartment of the thigh and is innervated by the _________________________ nerve.

hip adductors, obturator

patella alta is also associated with

hoffa fat pad impingement, typically observed during full flexion of the knee

proteoglycans can imbibe and retain large amounts of water, giving the nucleus it hydrodynamic properties, if the nucleus pulpopsus loses it proteoglycans, it can no longer

hold its water and no longer properly brace the annulus firbrosus from collapse

the time interval or sequence of motions occurring between two consecutive initial contacts of the same foot with the ground

human gait cycle

The implant to treat talar AVN received approval through the FDA's ___ a pathway for devices intended to benefit patients with a disease or condition that doesn't affect more than 8,000 people annually in the United States...

humanitarian device exemption process,

A rare, inherited disease that causes the progressive breakdown of nerve cells in the brain. symptoms can develop at any time, but often first appear when people are in their 30s or 40s

huntingtons disease

turf toe vs sand toe MOI

hyperextension injury vs hyper flexion injury

most common MOI of anterior cruciate ligament

hyperextension, large valgus or varus producing forces with foot planted, and either of the above combined with large rotational (torsional) force at the knee

Atypical slipped capital femoral epiphysis (SCFE) commonly occurs in overweight or obese boys who also have ____________________________. (Read this question carefully and indicate all the correct answers).

hypogonadism hypothyroidism

etiology of hallux values

idiopathic

etiology of scheuermann's disease

idiopathic but is present in approximately 10% of the population

acetabular labral tears

idiopathic, repetitive activities, trauma, diseases or conditions that damage the labrum (ex hip osteoarthritis, combined FAI)

Prime movers for hip flexion (there are 10 hip flexion muscles total)

iliac, psoas, rectus femoris, and sartorius

what are the prime movers for hip flexion?

iliacus, psoas, rectus femoris, and sartorius

"The inverted Y"; limits hip extension (and hyperextension), superior fibers limit adduction, inferior fibers limit abduction

iliofemoral ligament

motor= internal clique and transversus abdominis and sensory= posterolateral gluteal region

iliohypogastric never L1

motor= internal oblique and transverses abdominis and sensory= upper middle thighs/ genital region

ilioinguinal nerver L1

a strong ligament with 5 bands, stabilizes the lumbar spine where it meets the sacral section of the pelvis, anchors the L5 vertebra on the pelvic grille, preventing it from sliding forward or from rotating

iliolumbar ligament

For the past two weeks, a 32-year old female recreational runner has had pain on the lateral aspect of her left knee near Gerdy's tubercle. She typically runs 3 to 5 miles per workout, 4 days per week, but hasn't increased or decreased her mileage for 4 months. On palpation, she is point tender in this area, but has full range of motion at both the left hip and knee joints. Her symptoms are most likely due to:

iliotibial band friction syndrome

Acetabulum = little vinegar cup

ilium, ischium, pubis

diagnostic ultrasound is becoming increasingly popular for

imaging disease and dysfunction

any disorder that disrupt or degrades the proteoglycans of the nucleus pulpous will

impair their water binding capacity and compromise the ability of the disc to restore and or maintain its height

Why is evidence based practice important?

improves the care delivered to our patients, provide tools for finding and evaluating the evidence, promotes critical thinking, and having evidence of efficacy of care will facilitate 3rd party reimbursement for your services

Myelomeningocele is repaired ____________, most babies will also have herniation of the brain into the base of the skull which leads to a buildup of extra fluid in the brain (hydrocephalus)

in utero or in the first one to two days of the patients life

acute and chronic compartment syndromes are both caused by ________, but present with very different clinical scenarios

increased intracomapartmental pressures

tibial stress fracture bone scan: technetium 99m whole body bone scan shows

increased tracer uptake in lateral aspect of mid shaft of the tibia

When performing a clinical assessment of a patient's posture, which of the following are anatomical landmarks commonly used to evaluate her/him for anatomical asymmetry? (Read carefully and be certain to indicate all correct answers).

lateral malleoli Answer greater trochanters patellae Answer inferior angle of the scapulae iliac crests

Which of the following factors is/are associated with the pathogenesis (onset) of osteitis pubis? (Please read this question carefully and identify all the correct answers).

increasing the intensity of an exercise program too quickly walking or running on hard surfaces such as sidewalks beginning an exercise program after a long period of physical inactivity

negative likelihood ration

indicates the likelihood that a negative test result was obtained in a person with the condition as compared to a person without the condition

Positive likelihood ratio

indicates the likelihood that a positive test results was obtained in a person with the condition as compared to a person without the condition

examples of non mechanical spine disorders

infection, neoplasia, ankylosing spondylitis

motor= gluteus maximus and sensory= none

inferior gluteal nerve L5, S1, S2

Osteitis pubis

inflammation of pubic symphysis, predominantly caused by repetitive contraction of the muscles that attach to the pubic bone and the pubic symphysis. Also associated with pregnancy, childbirth, gynecological and urological surgery. most at risk are distance runners, weightlifters ice skaters, and dancers

osteitis pubitis etiology

inflammation of the pubic symphysis, repetitive motion, overtraining

potential causes of hip pathology

inflammatory (19) metabolic (2) neoplastic (5) and vascular (5)

Femoral triangle base

inguinal ligament, orgin= ASIS insertion= pubic tubercle

during a single stride, there are 2 periods of double limb support:

initial and terminal

knee valgus angle at ________ and maximum valgus angle during stance was significantly higher in _______- high school basketball players

initial stance, females

Femoracetabular Impingement (FAI) - exam 2

insidious onset groin pain, characterized by decreased clearance and abnormal contact bw the femoral head-neck junction and the acetabular rim.

medial patellofemoral ligament is the primary restraint to - exam 2

lateral patellar translation

Which of the following are static stabilizers of the patellofemoral joint? (Please read this question carefully and identify all the correct answers).

lateral patellofemoral ligament trochlear groove of the femur medial patellofemoral ligament

trendelenburg ST for what?

lateral tilt based on glut med gait

keystone of transverse metatarsal arch

intermediate 2nd cuneiform

a benign, inflamed or swollen nerve in the foot that causes a sharp burning sensation and/or pain in the forefoot area. The most common location for this to occur is between the 3rd and 4th toes; however, it can also occur between the 2nd and 3rd toes.

intermetatarsal neuroma

Fluroscopy are real time radiographic pictures and movies during medical procedures to guide:

internal fixation for fractures and proper placement for spinal injection therapy

Visceral (gut) pain originates from

internal organs

dynamic stabilizers of the knee rotators

internal rotators: PES ANSERINUS, sartorius, gracilis, semitendinosus, SEMIMEMBRANOSUS, POPLITEUS external rotator: biceps femoris

patients with low back pain who could identify their pain generator, the sources included

intervertebral discs, facet joints, SI joints and the hip

subtler joint

invasion 0-20 degrees and eversion 0-20 degrees

calcaneofibular ligament limits excessive

inversion

frontal plane movement where the medial border of the foot moves toward the midline

inversion

hind foot movements

inversion 20 and eversion 10

lateral ankle sprains MOI

inversion or plantar flexion + inversion

the subtalar joint in the rear foot allows for the combined motions of

inversion/ adduction (supinations) and eversion/abduction (pronation)

internal coxa saltans "snapping hip syndrome"

involves iliopsoas tendon tightly gliding over the iliopectineal line of the pelvis or femoral head

limits hip extension and internal rotation

ischiofemoral ligament

temporal limitations of the human muscular response

it take 100-200 msec to react to a stimulus, an additional 100 msec is needed to initiate movement

static stabilizers of the knee

joint geometry, ligaments, menisci, negative joint pressure, compressive loads in closed kinetic chain

ACL predisposing factor based on anatomy

joint laxity, intercondylar notch size

Which of the following are anatomical motions produced by muscles in the pes anserine group? (Read this question carefully and be sure to indicate all the correct answers).

knee flexion Knee internal rotation

The _________________________ muscle group is located in the posterior compartment of the thigh and is innervated by the _________________________ nerve.

knee flexor, sciatic

evaluations of an ankle sprain starts at the

knee with a squeeze test

Osgood Schlatters disease prognosis

known as self limiting condition, it is an adolescent condition that will diminish as the patient matures

what nerve roots does the sciatic nerve arise from?

l4, l5, s1, s2, s3

The SI joint is a

large diarthrodial joint that connects the spine with the pelvis

tibiofemoral joint

largest diarthrodial joint in the body

primary stabilizers of deep lateral compartment of the knee

lateral (fibular) collateral ligament (FCL or LCL), popiteofibular ligament (PFL), and popliteus tendon

_______ comprise the vast majority of ankle sprains, with epidemiological studies indication ______ of ankle sprains_____

lateral ankle sprains, 85-90% occur on the lateral aspect

orthopedic issues associated with patients with large Q angles

lateral tracking of the patella outside of the intercondylar notch of the femur, irritation/ breakdown of the articular (hyaline) cartilage on posterior patella, and increased risk of lateral patellar subluxation and or dislocation

Lordosis is also caused by

lax abdominal muscles, tight erector spine and hip flexor muscles, spondylosthesis and high heeled shoes

pronation-external rotation

leads to medial malleolus fracture (or deltoid ligament rupture) followed by rupture of the anterior inferior tibiofibular ligament, interosseous ligament, interosseous membrane, high fibula fracture then a posterior malleolus fracture

aging causes cells of most human tissues to become

less able to reproduce, less responsive to their environment and less efficient in the respective roads

pes anserine bursa

lies deep to the 3 tendons of the pes anserine

sex related neuromuscular imbalances are often observed in female athletes:

ligament dominance, quadriceps dominicane, and leg dominance

regional mechanical back pain, 90% of all cases includes

ligament sprains, muscle strains, facet joint osteoarthritis, sponylolysis, traumatic vertebral fractures, osteoporotic compression fractures, severe scoliosis

the yellow ligament, can stretch 70% of its resting length and not be damaged, its elastic properties assist muscles in returning the trunk from flexion to a neutral position

ligamentum flavum

Motion at the SIJ is _______ but interdependent on motion at the _______

limited, hip and lumbar spine

Fractures or dislocations of any of the tarsometatarsal joints, including:cuneiforms (3), cuboid, bases of the metatarsals (5)

lisfranc injuries

myositis ossificans traumatics pathology

localized formation of non neoplastic heterotypic bone and cartilage in the soft tissues in or adjacent to muscle and proximity to bone

In patients with patella alta, the length of their patellar tendon is significantly _________________ than the length of their patella, which in turn _____________________ the patellofemoral contact forces at the knee.

longer, increases

the knee going unites the 2 _________ in the body

longest lever arms (femur and tibia)

Which one of the following types of meniscal tears is most likely to be successfully repaired with arthroscopic surgery?

longitudinal

which type of meniscal tear is repairable

longitudinal tear

at 90 degrees of flexion the Posterolateral bundle is ____ and the anteromedial bundle is ____

loose and taut (opposite at 90 degrees of extension)

in the severely degenerated disc, the nucleus has

lost it ability to cushion the load which can lead to disc herniation

__ is the most prevalent of all musculoskeletal conditions, and most common form of musculoskeletal pain reported by adults 25 yr and older

low back pain

postural muscles become facilitated and tight (iliopsoas, rectus femurs, and erector spine) and phasic muscles become weak and inhibited (rectus abdominis, transverse abdonminis, gluteus Maximus and gluteus medium)

lower crossed syndrome

tight erector spine and iliopsoas , weak gluteus Maximus and abdominals

lower crossed syndrome

provides motor and sensory contributions to the anterior and medial compartment of the leg as well as to the abdominal wall and pelvic areas

lumbar plexus

According to Reiman (2016), scoliosis is a postural dysfunction that can involve which of the following? (Read carefully and be certain to indicate all correct answers).

lumbar region of the spine both the thoracic and lumbar regions of the spine (two curves) thoracic region of the spin

almost all chronic pain in the lower back occurs in a 6x4in area. Pain the lower back occurs in the area where the

lumbar vetebrae joint the sacrum and iliac crest

adult spinal anatomy

lumbar= 5 moveable vertebrae sacral= 5 fused vertebrae (adults) coccyxgeal= 4 fused vertebrae (adults)

results in 6 moveable lumbar vertebrae instead of the typical 5

lumbarization of S1

Assimilation of L5 to the sacrum is more common than ______ and occurs in approximately ___

lumbarization of the S1, 17% of the population

posterior drawer test

pos -- increased amt of post translation imp -- pcl sprain

According to Reiman (2016), which one of the following diagnostic imaging techniques is recommended for adolescent patients who present with low back pain and suspected stress reactions involving the lumbar pars interarticularis?

magnetic resonance imaging

. Which one of the following is the most appropriate diagnostic imaging technique to rule in/rule out a lateral meniscus tear in a 12-year old female soccer player?

magnetic resonance imaging (MRI)

According to Reiman (2016), which one of the following diagnostic imaging techniques is the most appropriate for identifying the presence of a herniated nucleus pulposus (HNP) correctly?

magnetic resonance imaging (MRI)

Which one of the following is the most appropriate diagnostic imaging technique to rule in/rule out an acetabular labral tear?

magnetic resonance imaging (MRI)

Recent studies have raised major concerns about the __________ associates with a CT scan,

magnitude of the dose of radiation, 100x more radiation than a standard x-ray

pathomechanic of tibial stress fractures: if the ___- and ____ of the loading exceed the bones ability to adapt, the injury occurs

magnitude, frequency

there is still no universally agreed upon protocol for the

management of an acute achilles tendon rupture

patella alta was associated with the highest

maximal pattelofemoral contact force and contact pressure

menstural cycle conclusion

may have an effect on anterior/ posterior laxity of the knee however, further investigation is need to confirm or reject

insall salvati ratio

measure patellar height, compares patellar tendon length to the greatest diagonal length of the patella

the hip adductors muscles are located in which compartment of the thigh?

med comp

what structures makes up the PCP (perimeniscal capillary plexus)

medial (inferior) geniculate artery lateral (superior) geniculate artery middle geniculate artery

erector spine arranged in 3 columns

medial = spinalis intermediate = longissimus lateral = iliocoatalis lumborum

deep tibiofemoral joint capsular ligaments

medial capsular ligament = anterior 1/3, middle 1/3, posterior oblique ligament lateral capsular ligament= anterior 1/3, middle 1/3, arcuate ligament (posterior)

arches of the foot

medial longitudinal arch, lateral longitudinal arch, transverse arch (midfoot) and metatarsal arch (forefoot)

mortise = notch

medial malleolus and lateral malleolus

soft tissue inflammatory condition with pain along the posteromedial border of the distal 1/3 of the tibia over a length of at least 5 cm. Often involves the periosteal attachments of the tibias posterior and soleus to the tibia

medial tibia stress syndrome (running induced injury)

patellar instability acute etiology

medial traumatic blow, lateral sublimation/dislocation, 50% recurrence rate in younger population, medial patellofemoral ligament is key

superficial tibiofemoral joint capsular ligaments

medial= tibial collateral ligament (TCL or MCL) lateral = fibular collateral ligament (FCL or LCL)

quadriceps angle norms

men 10 degrees or less, women 15 degrees or less

The anomaly in which there is a posterior defect in the spine that allows the protective coverings of the spinal cord to form a fluid-filled sac outside the body, spine is known as ________________________.

meningocele

moderate form of spina bifida in which only a fluid filled sac herniates outside the spine and the spinal nerves do not. Usually there is little to no nerve damage but can cause minor disability. May require early repair after birth or in the first 6 months of the patients life depending on the skin overlying

meningocele spinal bifida

most serious form in which the nerves herniate through the bony defect outside of the body and are covered bu a thin fluid filled sac. The nerve that are exposed to the amniotic cavity are damaged over time leading to difficulty walking and problems with bladder and bowel function

myelomeningocele spina bifida

disease of spinal cord, possible symptoms include stiffness, incoordination, weakness and numbness

myelopathy

bakers popliteal cyst

names after William mordant baker, swelling caused by synovial fluid protruding into the popliteal area, when excess knee joint fluid is compressed by body weight, it can become trapped and separate from the joint to form a fluid filed sac

position of maximal fluid holding capacity of the tibiofemoral joints synovial cavity

natural splinting position, 15-25 degrees

keystone of medial longitudinal arch

navicular

midfoot bones

navicular, 3 cuneiforms and cuboid

disease of peripheral nerve, possible symptoms include weakness and numbness

neuropathy

if kappa or ICC measure is 0-.1, 0.11-0.40, 0.41-0.60, 0.61-0.80, 0.81-1.0

no agreement, slight, fair, moderate, substantial agreement

There is __ between the two sesamoids and the Flexor Hallucis Longus (FHL) tendon that passes between them.

no direct connection

MRI pros

no ionizing radiation, enables 2D and 3D imaging of all tissues, appropriate for imaging all tissues, T1 and T2 weighting creates contrast between different types of tissue and fluid

distal tibiofibular syndesmosis

no voluntary movement

dynamic valgus alignment criteria - exam 2

non contact, extended hip/knee positions, valgus collapse, int tib rot, pronated foot

nonsurgical treatment of syndesmosis sprains

non weight bearing cast for 2-3 weeks, rocker boot or brace that prevents external rotation is needed for several weeks, typical clinical course until return to play/ work is 8-12 weeks

most clinical tests of the meniscus are preformed in

non weight bearing positions despite most meniscal symptoms manifesting during weight bearing

open kinematic chain

non weight bearing situations, the tibia externally rotates on the fixed femur

posterolateral drawer test

pos -- increased external rotation of the lateral tibial condyle imp -- damage to PL corner an PCL

a biomechanics process that causes unaviodable age related stiffening in the tissues that have a high collagen content and low metabolic rate. These stiffened tissues have reduced ability to absorb strain energy and become more vulnerable to damage from mechanical impacts

non-enzymatic glycation

the more avascular the tissue the more severe the

non-enzymatic glycation ruction

DEXA clinically diagnosis

normal bone density, osteopenia, and osteoporosis

stages of disc herniation

normal, degeneration, prolapse, extrusion, sequestration

Kellgren Lawrence Classification for osteoarthritis using radiographs

normal= no features of OA doubtful= minute osteophyte mild=definite osteophyte but normal joint space moderate= moderate joint space reduction severe= joint space greatly reduced, subchondral sclerosis

Eges test

not for patients suspected of having acute meniscal tears, rated as having moderate clinical value, 67% and 64% sensitivity, 81% and 90% specificity

hypothesis 3: obesity related differences in relative effort would be larger among older women that young women

not supported, women who were obese exerted greater relative effort while walking regardless of their age

a hydrated gel located in the center of the disc that when compressed expends in a radial fashion (resisted by the annular lamellae)

nucleus pulposus

comprised largely of proteoglycans, large molecules of complex sugars and protein and water (60-70%)

nucleus pulposus

intevertebral discs

nucleus pulposus and annulus fibrosus

Reiman (2016) suggested that the main function of plain film radiography ("x-rays") is to rule out fractures and conditions such as _______________________________________. (Read carefully and be certain to indicate all correct answers).

osteoarthritis exostoses neoplasms

In Chapter 20, Reiman (2016) notes that pelvic girdle pain (PGP) is a condition that occurs in patients who present in clinic with which of the following conditions? (Read this question carefully and indicate all the correct answers)

obesity rheumatoid arthritis osteoarthritis second or third trimester of pregnancy

Which of the following is/are considered a primary anatomical component of the posterolateral corner (PLC) of the tibiofemoral joint? (Read this question carefully and be sure to indicate all the correct answers).

oblique popliteal ligament fibular collateral ligament popliteus tendon popliteofibular ligament

cohort studies, case control studies and case reports

observational studies are single studies Level 3,4,5 C

Primary goal of an examination

obtain clinical diagnosis (another goal is to obtain sufficient info to determine a plan of care to improve pts quality of life)

6 deep external hip rotators

obturator interns, obturator externus, quadrates femoris, piriformis, Gemellus superior and Gemellus inferior

motor= obturator externes, adductor Magnus, longue and breves, pectiques, gracilis and sensory= medial thigh

obturator nerve L2. L3. L4

conditions that warrant termination of the orthopedic evaluation during observation

obvious fracture or joint dislocations and cyanosis

3 types of spina bifida

occulta, meningocele, myelomeningocele

Legg-Calve-Perthes Disease

occurs bilaterally in about 10-20% of affected children

visceral pain

often described as generalized aching or squeezing pain, common sources include IBS, period cramps, cancer, endo, prostatitis

PCL MOI

often from fall on flexed knee, blow to anterior knee, posterior loading PCL provides 90% of the resistance against posterior tibial translation

Patella Alta (high riding) effects

onset of tendo femoral contact is delayed, patellofemoral contact force increases with increasing knee flexion angle, the patients patella tends to sublux

Patella infera or baja (low riding) effects

onset of tendo femoral contact occurs early at relatively low knee joint flexion angles

Ligaments and capsular structures are slack. Joint surfaces may be distracted several mm. Theres positions allow the necessary motions of spin, roll and glide and may decrease joint function

open packed positions

the fibers in each successive lamella are oriented in the

opposite direction from the other

Iliotibial band

origin= iliac crest insertion= lateral tibia at gerdys tubercle

medial patellofemoral ligament anatomy

origin= medial adductor tubercle, proximal to the insertion of the MCL and distal to the insertion of the adductor Magnus tendon insertion=proximal 2/3 of the medial border of the patella

patellar chondromalacia

pain in anterior knee, typically under the patella, pain worsens with stairs and other knee flexion activities, crepitus present use to a roughening or softening of the patellar articular cartilage

Which of the following are included among the 33 "red flag findings" identified by Reiman (2016) that warrant immediate referral of the patient to physician? (Read carefully, and select all the answers that you consider to be correct).

pain that is not improved with therapy treatment night sweats increased urination with excessive thirst no discernible pattern of symptomsre bound tenderness

sports hernia symptoms

painful, immediately afterwords, pain localized over groin area; made worse by activity, relieved by rest

how to find the dorsals pedis pulse

palpate between the EHL and EDL tendons

how to find the posterior tibial artery (supplies 70% of blood to the foot)

palpate halfway between the medial malleolus and the achilles tendon

special tests used to evaluate osgood schlatters

palpate insertion of the patellar tendon into tibial tubercle, MMT of quadriceps (knee extension)

Compression of the S1 nerve root might manifest in the lower extremity as which of the following signs and/or symptoms? (Please read this question carefully and identify all the correct answers).

paresthesia in the posterior thigh, lateral lower leg and plantar aspect of the foot weakness during resisted knee flexion diminished Achilles tendon reflex

ACFAS grade 2

partial tear of ATF and stretching of CF

Total Musculoskeletal Assessment

patient history observation examination of movement special tests reflexes and cutaneous distribution joint play movements palpation diagnostic imaging

valgus stress test of MTP/IP joints

pos -- Pain or increased laxity imp -- Sprain of collat ligs within toes

observation and palpation for posture, lateral aspect

pelvic position (level of ASIS and PSIS on same side?)

trunk movement is coordinated with pelvic movement so that as the

pelvis rotates in one direction, the trunk rotates in the opposite direction and arm swing assists the trunk in rotation

other causes of low back pain <1% of all cases

pelvis, renal, abdominal aortic aneurysm, gastrointestinal, pagets disease

Patellofemoral pain syndrome (PFPS), sometimes referred to anterior knee pain syndrome (AKPS), is a prevalent condition among _________________________________________. (Please read this question carefully and identify all the correct answers).

people who also have chondromalacia of the patella Answer individuals who often have an imbalance between their vastus lateralis and vastus medialis muscles resulting in lateral tracking of the patella women who typically have Q-angles greater than 15 degrees people participating in repetitive motion activities such as bicycling and running.

varus stress test of MTP/IP joints

pos -- Pain or increased laxity imp -- Sprain of collat ligs within toes

fibular/ fibularis =

peroneal/ peroneus

evidence based practice is not a blueprint for how your must practice medicine, factors to be considered:

personal experiences, clinical judgement and patients preferences

Very rigid foot with an arch that sits higher from the ground. Well defined arch. Excessive pressure to rear foot and forefoot. Potential problems include plantar fasciitis, heel pain syndrome, arch sprain, metartarsalgia, calluses and claw toes

pes cavus (high arched feet)

Very flexible foot with an arch that sits low to the ground. Potential problems include plantar fasciitis, post tibial tendonitis and heel spurs. Low arches are more flexible and tend to roll inwards and over pronate

pes planus (flat feet)

A biomechanically efficient, moderately flexible foot with a defined arch. Susceptible to common foot problems such as heel pain and metatarsalgia from repetitive stress and improper fitting footwear

pes rectus (medium height arch)

sciatic nerve compression can lead to what disorder?

piriformis syndrome

Apophyseal avulsion diagnostic imaging

plain films (x rays) for fractures or MRI for soft tissue injuries

A broad aponeurosis of inelastic connective tissue. Protects neruvascular structures on plantar aspect of foot. Runs from the calcaneus to the heads of each of the 5 metatarsals

plantar fascia

A sharp pain or burning sensation when first putting weight on the foot, particularly in the morning, i.e., the patient's first steps of the day.Stiffness on the bottom of the heel (at calcaneal origin). An ache in the medial or lateral longitudinal arch

plantar fasciitis symptoms

talocural joint movements

plantar flexion 50 and dorsiflexion 20

supination 45-60 is a combination of

plantar flexion, hindfoot inversion and forefoot adduction

A patient who presents in your clinic with a calcaneal gait has weak or paralyzed _________________ muscles.

plantar flexor

functions of the superficial posterior tibial compartment muscles

plantar flexors -gastrocnemius (medial and lateral head) -soleus (deep) -plantaris

functions of the deep posterior compartment muscles

plantar flexors (all 3) and toe flexors (flexor digitorum longus and flexor hallucicus longus)

Following an acute left quadriceps femoris strain, a 20-year old intercollegiate track sprinter is unable to extend his knee actively beyond 30 degrees without assistance from the examiner. Using the manual muscle test (MMT) grading format presented below, you would classify his quadriceps muscle strength as:

poor = grade 2

Regardless of open kinetic chain (OKC) or closed kinetic chain (CKC) positioning, the ___________________ unlocks the extended tibiofemoral joint by initiating knee flexion and rotation.

popliteus

deep posterior tibial compartment muscles

popliteus, tibialis posterior, flexor digitorum longus and flexor hallucis longus

posterior sag sign

pos -- - posterior displacement of tib tuberosity imp -- pcl tear

Supple Pes Planus (Windlass) Test

pos -- 1) if arch collapses when they stand 2) sharp pain when standing up imp -- 1) supple pes planus 2) plantar fascitis

intermetatarsal glide test

pos -- Pain or increased glide/laxity imp -- damage to the deep transverse MT and/or interosseous ligament

who is the #1 at risk population for ACL injuries

post menarche female athletes (12-15)

If a positive Godfrey test is observed, which one of the following structures has been damaged to some degree?

posterior cruciate ligament

Which one of the following is considered to be the "strongest" (greatest tensile strength) ligament in the knee?

posterior cruciate ligament

motor= none and sensory= posterior thigh and leg, perineum

posterior femoral cutaneous nerve S1, S2, S3

thickest at C spine, things inferiorly and limits flexion

posterior longitudinal ligament

A herniated nucleus pulposus (HNP) in the lower lumbar region that is extruded or sequestered in the __________________ direction most often results in unilateral sciatica.

posterolateral

the most common herniated nucleus pulposus

posterolateral (foraminal)

Which of the following types of information would typically be obtained by the clinician during the "O" phase of the HOPS or SOAP injury examination process?

posture gait both gait and posture muscle atrophy ALL THE ABOVE

You have a 46-year old male patient who is employed as a tile floor installer for a large national company. He has come to your clinic about the pain and swelling he now has on the anterior aspects of both of his patellae. From your history you learn that he doesn't wear any knee pads or use a kneeling cushion while at work. You suspect he has a case of chronic bilateral bursitis at the knee, most likely involving the _____________________ bursa.

prepatellar

what is the most common bursitis?

prepatellar bursa

Which of the following represent information that would typically be obtained by the clinician from the patient during the "O" phase of the HOPS injury examination process? (Read carefully and be certain to indicate all correct answers).

presence of surgical scars impaired gait skin color or appearance fascial expression asymmetry soft tissue contours

Which of the following are risk factors associated with slipped capital femoral epiphysis in an adolescent patient? (Please read this question carefully and identify all the correct answers).

previous family history of this condition metabolic disorders body weight above the 95th percentile endocrine disorders

intrinsic risk factors for tibial stress fractures

previous injury, anatomic malalignments, muscle strength and flexibility and low bone mineral density

the ACL is the _____________ (about 85%) to anterior tibial translation

primary restraint

The curve of the sacral spine is a ___________________ curve in the _____________________ direction.

primary, posterior

The curve of the thoracic spine is a ___________________ curve in the _____________________ direction.

primary, posterior

For the majority of patients who elect not to have ACL reconstructive surgery, the "natural history" of their injury includes which of the following? (Please read this question carefully and identify all the correct answers).

progressive deterioration of the knee joint meniscal tears osteoarthritis

Which of the following have been recognized as YELLOW flag findings that warrant referral of a patient to a physician? (Read carefully and be certain to indicate all correct answers).

progressive weakness symptoms that are peripheralizing multiple inflamed joints bilateral symptoms

somatic pain can be either

superficial (skin or mucus membranes) or deep (bones, joints, tendons)

pathomechanics of tibial stress fractures: bone responds to _________ by adapting its structure in accordance with _______

repetitive loading,Wollfslaw

conditions that warrant terminations of the orthopedic evaluation during history

reports the inability to feel or move one or more limbs, complaints of significant chest pain and complaints of significant difficulty in breathing

anterior cruciate ligament

resists anterior translation of the tibia relative to a fixed femur or posterior translation of the femur relative to a fixed tibia, resists the extremes of knee extension, resists valgus and various deformations and excessive horizontal plane rotations

posterior cruciate ligament

resists posterior translation of the tibia relative to a fixed femur or anterior translation of the femur relative to a fixed tibia, resists the extremes of knee FLEXION, and resists valgus and varus deformations and excessive horizontal plane rotations

medial collateral ligament

resists valgus deformation of the knee and resists excessive knee extension

lateral collateral ligament

resists varus deformation of the knee, resists excessive knee extension

in comparison to MRI, diagnostic ultrasound produces greater ______, requires less _____ and

resolution, imaging time, costs less

exertion tibial compartment compression syndrome symptoms:

resolve relatively soon (<30min) after the activity is ceased

pathomechanics of tibial stress fracture: this remodeling process includes ______ of bone where not needed, and _______ of new bone in areas of new mechanical loading

resorption, deposition

supination-adduction

results in a transverse fracture of the distal fibular than an oblique fracture of the medial malleolus

when surgery is performed for moderate to severe syndesmosis injuries, the prognosis is excellent following DRIF with a screw or suture button reduction of the syndesmosis

return to sports may take 4-6 months

zygoapophyseal (facet) joints

right superior articular, left superior, right inferior, left inferior

A secondary ossification center of the vertebral endplate connected to the intervertebral disc, firmly attached to the annulus fibrosis through sharpeys fibers and its ossification begins at 4-6 yrs old

ring apophysis

rolling vs sliding

rolling is predominant at the initiation of flexion, sliding occurs more at the end of flexion

CT scans are similar to x-rays but they __________ to create a series of 2D and #D computer generated images

rotates 360 degrees around the body part

of the lumbar spine is achieved by each disc allowing a small degree of torsion and is resisted by tension developed within the annulus fibrosus

rotation

if a patient does not report pain 2cm-6 above the insertion of the achilles, this has a fair ability to ________. If a patient does report pain then it is fairly helpful to ________

rule out achilles tendinopathy, rule out

ACFAS grade 3

rupture of ATF, rupture of CF, partial tear of PTF and partial tear of TF

provides motor and sensory nerves for the posterior thigh, most of the lower leg, entire foot and part of the pelvis

sacral plexus

results in 4 moveable lumbar vertebrae instead of 5

sacralization of L5

condyloid joint moves in the

sagital place (flexion and extension) and horizontal/ transverse plane (internal/external rotation, no rotation occurs at 0 degrees extension, maximum rotation occurs at 90 degrees)

Apophyseal avulsion fracture pathology

salter Harris type 1 fracture of an apophyseal cartilage plate

femoral triangle lateral border

sartorius

Which of the following muscles, when acting concentrically, flex the hip? (Please read this question carefully and identify all the correct answers).

sartorius rectus femoris psoas iliacus

motor (tibial)= hamstrings, gastrocnemius, soles, plantaris and motor (fibular)= anterior and lateral compartment m of leg and sensory (tibial)= posterolateral and medial foot, sole of foot and sensory (fibular)= anterolateral leg, dorm of foot

sciatic nerve L4, L5, S1, S2, S3

the most common etiology of ____ is herniated nucleus pulposus, typically neuropathy and usually only affects one leg

sciatica

one or more excessive lateral curvatures of the thoracic or lumbar spine or in both

scoliosis

_______ postures apply more pressure to the disc than do standing and recumbent position which explain the exacerbation of symptoms of herniated disc when patients are

seated/bending, in seated and/or forward flexed positions

The cervical curve is a ______________________ curve that begins to develop ________________.

secondary, at 3 months of age

The lumbar curve is a _____________________ curve that develops as the result of _________________________.

secondary, the infant progresses from sitting to crawling to walking

medial meniscus anatomy

semilunar (C), narrow anteriorly, connected to TCL

which of the following act as an agonist to the ACL and helps resist ant tib trans - exam 2

semitendonosus, semimembranosus, biceps femoris, med meniscus, lat meniscus, screw home mechanism

Irritation or fracture of the 2 small bones on the plantar surface of the "big toe" (hallux).Signs and Symptoms: Pain on the "ball of the foot" (plantar aspect of great toe), Edema and/or ecchymosis, Difficulty flexing or extending thegreat toe.

sesamoiditis

An AVULSION FRACTURE of the tuberosity of the 5th metatarsal is typically caused by a

severe inversion sprain that pulls (avulses) the fibularis brevis tendon from its insertion site.

Which of the following types of information can be obtained with a dual energy X-ray absorptiometry (DEXA) scan? (Read carefully and be certain to indicate all correct answers).

severity of osteoporosis body fat percentage presence of osteopenia bone mineral density

increased pronation occurs in the subtler joint of the long leg, accompanied by a shift of the trunk toward the longer extremity

short leg gait, caused by true anatomical leg length discrepancy

observation and palpation for posture, anterior aspect

shoulder heights, iliac crest heights, ASIS heights, patellar positions, fibula head heights, lateral malleoli

3 key factors that create need for non-musculoskeletal related pain

side effects of medications Comorbidities Visceral pain mechanisms

3 key factors in the patients screening process

side effects of medications, comorbidities, visceral pain mechanisms

Reiman (2016) describes the patient screening process for non-musculoskeletal related pain and dysfunction as complicated and time consuming. That said, which of the following is/are key factors that create the need for this screening include? (Read carefully and be certain to indicate all correct answers).

side effects of prescribed medications comorbidities the presence of visceral pain

Neuroimaging is not necessary in a child with a __________ because CT is associated with radiated exposure that ____________, MRI is also associated with risks from _______

simple febrile seizure, may escalate future cancer risks, required sedation and high cost

Myositis ossificans traumatics etiology

single episodes of severe trauma or repeated episodes of minor trauma

traumatic injury to the lateral ankle or overuse are the main causes, the pain is most severe when standing, walking on uneven ground, or during the movements of supination and adduction of the foot

sinus tarsi syndrome

Due to the incongruence in length of the femoral condyles compared to the tibial condyles, the arthrokinematics of the tibiofemoral joint include which of the following? (Please read this question carefully and identify all the correct answers).

sliding (gliding) Rolling

obesity is linked to a number of gait alterations

slower gait speed, shorter step length, longer duration stance and double support phases, and difficulty walking long distances

test for sciatica

slump test

When you perform a special ligament test on an injured patient with an acute Grade 2 sprain of her knee, the quality of the end feel you'll obtain is typically described as ________________

soft

grade 2 sprain

soft end feel, significant instability, 5-10mm more laxity than the patients contralateral joint

In the basic systematic "ABCs" approach to evaluating conventional radiographs described in Reiman (2016), the "s" in the mnemonic represents ________________________.

soft tissue

the gold standard for direct measurement of tibial compartmental pressures

solid state transducer inter compartmental catheter (STIC)

a condition that affects the spine and is usually apparent at birth, it is a type of neural tube defect

spina bifida

the mildest type of spina bifida. Sometimes called hidden spina bifida. There is a small gap in the spine but no opening or sac on the back. Many times it is not discovered until late childhood or adulthood but it does not usually cause any disabilities

spina bifida occulta

defined as the forward translation of one vertebral body on another, required the bilateral fracture or non union of the right and left pars interarticlaris of the same vertebrae. most commonly occurs in the lower lumbar spine (L5-S1 most common, L4-L5 second most common and so on)

spondylolisthesis

Which of the following are known causes of regional mechanical low back pain that also results in neurogenic leg pain? (Read carefully and be certain to indicate all correct answers).

spondylolisthesis spinal stenosis herniated nucleus pulposus

present in about 3-6% of the gen population, unilateral lesion fx of the pars interarticularis (isthmus), about 20% of pars interarticluaris defects are identified on lateral oblique x rays

spondylolysis

MOI: running on hard surfaces, improper footwear or overuse. Symptoms: pain, difficulty walking/running, discoloration over plantar surface

sprain of the transverse and longitudinal arch

Sand Toe is a___ After initial treatment with the RICE protocol, patients are sometimes immobilized in a walking boot or cast. Surgery is almost never required for Sand Toe unless other injuries have occurred.

sprain of the volar (dorsal) ligaments of first MTP joint ("big toe") that is commonly seen in beach volleyball.

the knee is a mechanical compromise

stability in extension and mobility in flexion

human gait cycle is divided into 2 phases

stance phase (60-65% of walking) and swing phases (35-40% of walking)

gait cycle percentages when jogging vs running vs sprinting

stance= 33%, swing = 30% for jogging stance= 30%, swing =30% for running stance=26%, swing=30% for sprinting

training methods

start low and go slow

major points regarding palpations

start on uninvolved side, no surprises, compare bilaterally

distance from heel strike of one foot to heel strike of the other foot in one gait cycle

step length

With regard to pathological gait patterns presented in Lecture 6, during swing phase of ______________________ , the affected limb demonstrates increased knee and hip flexion to avoid toe dragging, producing a "high-step" pattern.

steppage gait

the foot slaps the ground at initial contact, owing to foot drop. During swing phase, the affected limb demonstrates increased hip and knee flexion to avoid toe dragging, producing a high step pattern

steppage gait (drop foot) caused by weakness or paralysis of the dorsiflexors

lateral tibial compartment nerve and artery

superficial fibular nerve and fibular artery

salter harris type 1 - exam 2

straight across / separated metaphysis and epiphysis, apophyseal avulsion fracture at the hip/pelvis

multicomponent ACL injury prevention programs typically should include at least 3 of the following types of exercise

strength, plyometrics, agility, balance, flexibility

progression of tibial stress fracture if not treated

stress reaction, stress fracture, frank (complete) fracture

ACFAS grade 1 for lateral ankle sprain

stretching of ATF and stretching of CF

distance from heel strike of one foot to heel strike of the same foot in one gait cycle

stride length

distance between the midline of one foot at mid-stance and the midline of the other foot at mid-stance

stride width

two types of scoliosis

structural (genetic, congenital "at birth", idiopathic) and non-structural (poor posture, leg length discrepancy, activities of daily living)

an actually difference in the length of the femur or the tibia of one leg compared to the other, possible causes: possible from disruption in the growth plate of one of the long bones or a congenital anomaly

structural or true leg length (naval)

Meta-analyses and systematic reviews

studies of studies that use data from multiple studies Level 1 A

the portion of the vertebral body to which the cartilaginous endplate is applied lacks a formal name, can be referred to as the

subchondral bone part of the vertebral body

typically occurs from damage to the perineal retinacula that are injured during an inversion ankle sprain

sublimating fibularis (peroneal) tendons

consists of the talus and calcareous, anatomical motions are inversion and eversion

subtalar joint

the foot position when the subtalar, talonavicular and calcaneocuboid joints are reduced and congruous

subtalar joint neutral position

dermatomes of the lower leg (all innervated by the sciatic nerve)

sural nerve, deep fibular nerve, superficial fibular nerve and medial calcanea branches

baker's cyst moi

swelling caused by synovial fluid protruding in the popliteal area, the fluid can become trapped

defined by the foot not being in contact with the ground ( 100% non weight brewing) and moving forward

swing phase

Which of the following are possible causes of effusion at the tibiofemoral joint? (Please read this question carefully and identify all the correct answers).

synovitis bacterial infection, e.g., MRSA Grade III ACL injury hemarthrosi

key injury prevention program design factors

takes less than 15 min to perform, 2-3 days/week, best results come with pre season and in season, includes sport specific exercises, includes variety and progression over time, and multicomponent models are the most common

clinically, the subtalar joint is neutral when the

talar head protrudes equally on the medial and lateral sides

many etiologies exist that can cause taller AVN, with the most common being

talar neck fractures

closed packed position = maximal contact of the articulating surfaces

talcrual joint= full dorsiflexion and subtler joint= full inversion or eversion

open pack positions = least stable positions

talocrual joint= 10 degrees of plantar flexion and subtler joint = neutral position

tenon = peg

talus

rear foot (hind foot) bones

talus and calcaneus

once talar collapse develops, surgical treatment typically involves an arthrodesis (surgical immobilization of a joint by fusion of adjacent bones) or

talus arthroplasty

7 tarsal bones

talus, calcaneus, navicular, medial (1st) cuneiform, intermediate (2nd) cuneiform, lateral (3rd) cuneiform, and cuboid

chopart joint

talus-calcaneus joint and navicular-cuboid joint

compression of the tibial nerve and posterior tibial artery as it passes through the tarsal tunnel on the medial aspect of the ankle

tarsal tunnel syndrome

More severe cases of Turf Toe will

tear the plantar plate of the MTP joint, along with the joint capsule

Which of the following diagnostic imaging techniques delivers ionizing radiation to the patient when the image is being obtained? (Read carefully and be certain to indicate all correct answers).

technetium-99 bone scan fluoroscopy

chronic tendinitis can result in a breakdown of the tendon or ______ reflecting a weakened tendon that is _____

tendinosis/ tendinopathy, easier to rupture

patellar tendinitis/ tendinopathy etiology

tendons have poor blood supply= slow to repair, overtraining or too rapid increases in training volume, inadequate stretching. Chronic, excessive loading can lead to partial tearing of the patellar tendon

3 forms of achilles tendinitis

tendynovitis= inflamed synovial sheath paratendinitis= inflamed paratenon sheath that surrounds the achilles tendinosis= microscopic tears of the achilles

a thin, triangular muscle that forms part of the floor of the popliteal fossa. Insignificant as a knee flexor but does unlock the knee from full extension. Assists in pulling the lateral meniscus posteriorly in knee flexion

the role of the popliteus

spina bifida might cause physical and intellectual disabilities that range from mild to severe, the severity depends of :

the size and location of the opening in the spine and whether part of the spinal cord and never are affected

what the clinician sees during loading response

the stance foot is flat on the ground

posterior cruciate ligament (PCL)

the stronger ligament in the knee, provides 90% of resistance to posterior tibial translation

Pathology

the study of the nature and cause of disease which involved changes in structure and function

Since the intervertebral disc is the largest avascular tissue in the body, the glycation process thrives within its substance. NEG results in a slow but steady transformation of the disc collagen into a

thicker, more brittle substance.

primary postural anomaly associated with upper and lower crossed syndrome

thoracic hyperkyphosis (upper) and lumbar lordosis (lower)

two primary posterior curves

thoracic= forms due to fetal position in womb sacral= kyphotic curvature remains throughout life

anterior tibial compartments muscles

tibial anterior, extensor digitorum longus, extensor hallus longus (TDH) and fibularis Tertius

Which one of the following knee ligaments is the primary restraint against valgus stress that is applied to a knee that is flexed to 30 degrees?

tibial collateral ligament

restraints to valgus force at 5 degrees knee flexion

tibial collateral ligament = 57% posterior oblique ligament= 18% deep capsular ligament= 5%

The "unhappy triad" injury at the knee involves which of the following combinations of anatomical structures?

tibial collateral ligament, medial meniscus, anterior cruciate ligament

restraints to valgus force at 25 degrees knee flexion

tibial collateral ligament= 78% posterior oblique ligament= 4% deep capsular ligament= 4%

_____- are the most common cause of acute tibial compartment syndrome

tibial fractures

supplies both the deep and superficial posterior compartments

tibial nerve

deep posterior tibial compartment nerve and artery

tibial nerve and posterior tibial artery

superficial posterior tibial compartment nerve and artery

tibial nerve and posterior tibial artery

Microfracture of cortisol bone. Commonly not visible on plain x-rays for up to 3 weeks. Focal hot spot on technetium 99m bone scan

tibial stress fracture (running induced injury)

acute pain on bone/ periosteum in response to exercise. Precursor to a stress fracture if not recognized and treated properly. Must modify activity level

tibial stress reaction (running induced injury)

dorsiflexion of talocrural joint anterior tibial compartment muscle degrees

tibialis anterior = 1 degree extensors digitorum longus = 2 degrees extensor hallucis longus = 2 degrees

talocrural joint movements in the sagital plane = dorsiflexion

tibialis anterior, EDL, EHL, fibularis tertius (TDH)

mechanoreceptors in the ACL are thought to act as _____- to convert the mechanical load on the ACL to afferent (sensory) nerve impulses

transducers

red on white zone

transitional zone 3mm to 5mm from emniscosynovial junction, intermediate healing potential

as the disc undergoes degeneration, the nucleus loses some of its cushioning ability and

transmits the load unequally to the annulus

iliolumbar ligament orgin and insertion

transverse process of L5 and lateral ilium

With regard to standing posture, Janda's lower crossed syndrome involves weakness and/or inhibition of which of the following muscles? (Read carefully, and select all the answers that you consider to be correct).

transversus abdominis rectus abdominis gluteus maximus

* CATEGORIES OF POSSIBLE HIP PAIN

trauma (17) Infections (5) Neurologic (3) Degenerative (2) Inflammatory (19) Metabolic (2) Neoplastic (5) Vascular (5) 58 total

patients exhibits an excessive lateral list in which the thorax is thrust laterally during the stance phase. This serves to maintain the center of gravity and prevent a drop in the pelvis on the affected side

trendelenburg gait caused by weakness of the gluteus medius

specificity

true negative rate, the tests ability to correctly detect those patients who do not have the disorder/ injury

sensitivity

true positive rate, the tests ability to detect those patients who actually have the disorder

the long articulating surface of the femoral condyles is about _____ the length of the tibial condyles

twice, because of this the condyles execute both rolling and sliding motions

most Danis-weber AO type A and B fractures were caused by low-velocity loading like ______ however, some type A and C fractures were caused by high velocity impacts like____

twisting, simple falls, falling down a flight of stairs. traffic accidents, falls from heights, sporting injuries

S-A-L-T-ER

type 1 (Straight across), type 2 (Above), type 3 (Lower), type 4 (Through), type 5 (pERasure of growth plate/ crush)

associated comorbidities with lumbar disc herniations

type 11 diabetes, hyperlipidemia, smoking

Danis-weber AO classification system. Three fracture types are based on the level of fibular fracture and the associated level of syndesmosis disruption

type A= transverse fibular fracture at or below ankle joint with no syndesmotic injury type B= spiral fibular fracture beginning at the level of the joint and a partial syndesmotic injury type C= a fracture proximal to the ankle joint with an associated disruption of the syndesmosis to this level

slipper capital femoral epiphysis (SCFE)

typical: idiopathic atypical: endocrine disorders, metallic disorders, radiation therapy -Grades of severity: mild (1/3 slippage) moderate (1/3-1/2 slippage) and severe (>1/2 slippage)

somatic pain

typically goes away after the injury heels but can become chronic

. Which of the following are included among the 33 "red flag findings" identified by Reiman (2016) that warrant immediate referral of the patient to physician? (Read carefully and be certain to indicate all correct answers).

unusual menstrual cycle or symptoms unexplained weight loss pain unchanged by movement or rest chest pain with exertion neurological signs and symptoms

lateral ankle sprain prognosis

up to 50% of patients will experience chronic ankle pain. rehab and brace has a 90% treatment efficacy. for the 10% who fail, surgery (90% success)

postural muscles become facilitated and tight, phasic muscles become weak and inhibited. The crux of the problem is chronic hypertonicity on one side versus muscle weakness or inhibition on the other

upper crossed syndrome

tight trapezius elevator scapular and pectorals, weak rhomboids serrates anterior and deep neck flexors

upper crossed syndrome

peripheral nerves are combined structures formed by the ventral (anterior) rami of spinal nerve roots. Upper and lower extremity examples

upper= median nerve (C5-C8, T1) and ulnar nerve (C7-C8, T1) lower= femoral nerve (L2-L4) and sciatic nerve (L4-L5, S1-S3)

how to measure subjective pain

visual analog scale, numeric pain rating scale, McGill pain questionnaire

why are genetics such a big factor

we may inherit genes that produce inferior collage or proteoglycan in our intervertebral disc the mechanical failure of the annulus fibrosus or nucleus pulposus can then occur earlier in life and/or at lower magnitude loads

Lumbar lordosis is known to be caused by which of the following? (Read carefully and be certain to indicate all correct answers).

weak core (abdominal) muscles tight hip flexor muscles pregnancy obesity

The S1 nerve root compression symptoms may include

weakness in gluteal area, back of thigh, post aspect of leg and sole of foot

closed kinematic chain

weight bearing situations, the femur internally rotates on the fixed tibia

factors increasing the Q angle

width of hips, genu valgum, poorly positioned tibial tubercle, tibial rotation, and pes planus (flat feet)

Early in the stance phases of gait (foot flat + mid stance), the plantar fascia is relaxed, and most feet function as flexible shock absorbers. Later in stance, during propulsion, the plantar fascia tightens as the body weight is transferred forward, and the foot becomes an efficient rigid lever arm

windlass mechanism

radiography includes

x-rays, fluoroscopy and DEXA scan

is it possible that testosterone contributes to the ACls ability to withstand tensile loads

yes

is the base of the metatarsal where the fibularis brevis inserts. Avulsion fractures from the pull of this tendon and attached ligaments are characteristic. AVULSION FRACTURES

zone 1 of the 5th metatarsal

is at the metaphyseal-diaphyseal junction, distal to the cancellous (styloid) tuberosity. Fractures, called Jones fractures, are particularly susceptible to nonunion and malunion because this region of the bone has a tenuous blood supply. JONES FRACTURES

zone 2 of the 5the metatarsal

is the diaphysis and fractures are most commonly trauma or rotational injuries leading to spiral fractures. MID SHAFT STRESS FRACTURES

zone 3 of the 5th metatarsal

Purnell's 12 Domains for Cultural Competence

• Culture overview and communication • Communication • Family roles and organization • Workforce issues • Bio-cultural ecology • High-risk behaviors • Nutrition • Pregnancy and childbearing practices • Death rituals • Spirituality • Healthcare practices • Health care practitioner


Kaugnay na mga set ng pag-aaral

Chapter 6: Mitigating Security Threats

View Set

Leadership/ Management/ Delegation/ Prioritizing

View Set

US History I: Chapter 18 Homework

View Set

Psychology Exam 4 Practice Questions

View Set